Mechanics of

The Respiratory System 17 Bones and Muscles of the Thorax Surround the Lungs Pleural Sacs Enclose the Lungs Airways Connect Lungs to the External Environment The Airways Warm, Humidify, and Filter Inspired Air Alveoli Are the Site of Is High-Flow, Low-Pressure G a s L a w s Air Is a Mixture of Gases Gases Move Down Pressure Gradients Boyle’s Law Describes Pressure-Volume Relationships Ventilation Change During Ventilation During Ventilation, Air Flows Because of Pressure Gradients Inspiration Occurs When Alveolar Pressure Decreases Expiration Occurs When Alveolar Pressure Increases Intrapleural Pressure Changes During Ventilation and Elastance May Change in Disease States Surfactant Decreases the Work of Breathing Airway Diameter Determines Rate and Depth of Breathing Determine the Effi ciency of Breathing Gas Composition in the Alveoli Varies Little During Normal Breathing Ventilation and Alveolar Blood Flow Are Matched Auscultation and Assess Pulmonary Function

This being of mine, whatever it really is, consists of a little fl esh, a little breath, and the part which governs. — Marcus Aurelius Antoninus (C . E . 121–180)

Background Basics Ciliated and exchange epithelia Pressure, volume, fl ow, and resistance Pulmonary circulation Surface tension Colored x-ray of the lung Autonomic and somatic showing the motor neurons branching Velocity of fl ow airways.

600 Mechanics of Breathing

magine covering the playing surface of a racquetball court cavity to control their contact with the outside air. Internalization (about 75 m2) with thin plastic wrap, then crumpling up the creates a humid environment for the exchange of gases with the Iwrap and stuffi ng it into a 3-liter soft drink bottle. Impossible? blood and protects the delicate exchange surface from damage. Maybe so, if you use plastic wrap and a drink bottle. But the lungs Internalized lungs create another challenge, however: how to of a 70-kg man have a gas exchange surface the size of that plastic move air between the atmosphere and an exchange surface deep wrap, compressed into a volume that is less than that of the bottle. within the body. Air flow requires a muscular pump to create Th is tremendous surface area for gas exchange is needed to supply pressure gradients. More complex respiratory systems therefore the trillions of cells in the body with adequate amounts of oxygen. consist of two separate components: a muscle-driven pump and a Aerobic metabolism in cells depends on a steady supply thin, moist exchange surface. In humans, the pump is the muscu- of oxygen and nutrients from the environment, coupled with loskeletal structure of the thorax. Th e lungs themselves consist of the removal of carbon dioxide. In very small aquatic animals, the exchange epithelium and associated blood vessels. simple diff usion across the body surface meets these needs. Dis- Th e four primary functions of the respiratory system are: tance limits diff usion rate, however, so most multicelled animals 1 Exchange of gases between the atmosphere and the blood. require specialized respiratory organs associated with a circula- Th e body brings in O for distribution to the tissues and tory system. Respiratory organs take a variety of forms, but all 2 eliminates CO waste produced by metabolism. possess a large surface area compressed into a small space. 2 2 Homeostatic regulation of body pH. The lungs can alter Besides needing a large exchange surface, humans and 17 body pH by selectively retaining or excreting CO . other terrestrial animals face an additional physiological chal- 2 3 Protection from inhaled pathogens and irritating substances. lenge: dehydration. The exchange surface must be thin and Like all other epithelia that contact the external environ- moist to allow gases to pass from air into solution, and yet at ment, the respiratory epithelium is well supplied with de- the same time it must be protected from drying out as a result fense mechanisms to trap and destroy potentially harmful of exposure to air. Some terrestrial animals, such as the slug substances before they can enter the body. (a shell-less snail), meet the challenge of dehydration with be- 4 Vocalization. Air moving across the vocal cords creates havioral adaptations that restrict them to humid environments vibrations used for speech, singing, and other forms of and nighttime activities. communication. A more common solution is anatomical: an internalized respiratory epithelium. Human lungs are enclosed in the chest In addition to serving these functions, the respiratory sys- tem is also a signifi cant source of water loss and heat loss from the body. These losses must be balanced using homeostatic RUNNING PROBLEM compensations. In this chapter you will learn how the respiratory system Emphysema carries out these functions by exchanging air between the envi- ronment and the interior air spaces of the lungs. Th is exchange You could hear her whistling, wheezing breathing preceding her down the hall. “Diagnosis: COPD,” reads Edna Wilson’s is the bulk fl ow of air, and it follows many of the same principles patient chart. COPD—chronic obstructive pulmonary that govern the bulk fl ow of blood through the cardiovascular disease—is the name given to diseases in which air system: exchange is impaired by narrowing of the lower airways. 1 Flow takes place from regions of higher pressure to regions Most people with COPD have emphysema or chronic of lower pressure. bronchitis or a combination of the two. Individuals in whom 2 A muscular pump creates pressure gradients. chronic bronchitis predominates are sometimes called “blue bloaters,” owing to the bluish tinge of their skin (from low 3 Resistance to air fl ow is infl uenced primarily by the diam- blood oxygen levels) and a tendency to be overweight. In eter of the tubes through which the air is fl owing. contrast, patients with emphysema have been nicknamed Air and blood are both fl uids. Th e primary diff erence be- “pink puff ers.” They tend to be thin, have normal (pink) skin tween air fl ow in the respiratory system and blood fl ow in the coloration, and often breathe out through pursed lips, which circulatory system is that air is a less viscous, compressible mix- helps open their airways. More than 12 million people in ture of gases while blood is a noncompressible liquid. the United States have COPD. Its most common cause is smoking, and most people can avoid the disease simply by not smoking. Unfortunately, Edna has been a heavy smoker for 35 of her 47 years. The Respiratory System The word has several meanings in physiology ( Fig. 17.1 ). Cellular respiration refers to the intracellular reaction of oxygen with organic molecules to produce carbon

601 Mechanics of Breathing

consists of structures involved in ventilation and gas exchange EXTERNAL RESPIRATION ( Fig. 17.2 ): The respiratory and circulatory systems coordinate to move oxygen 1 Th e conducting system of passages, or airways , that lead and CO between the atmosphere and the cells. 2 from the external environment to the exchange surface of Exchange I: CO2 O2 the lungs. atmosphere 2 Th e alveoli (singular alveolus) {alveus, a concave vessel}, to lung a series of interconnected sacs and their associated pulmo- (ventilation) Airways nary capillaries. Th ese structures form the exchange sur- face, where oxygen moves from inhaled air to the blood, Alveoli of lungs and carbon dioxide moves from the blood to air that is CO2 O2 about to be exhaled. Exchange II: lung to blood 3 Th e bones and muscles of the thorax (chest cavity) and ab- O CO2 2 domen that assist in ventilation. Transport of Pulmonary gases in Th e respiratory system can be divided into two parts. Th e circulation the blood upper respiratory tract consists of the mouth, nasal cavity, pharynx, and larynx. Th e lower respiratory tract consists of the trachea, two primary bronchi { bronchos, windpipe; singular— bronchus}, their branches, and the lungs. Th e lower tract is also known as the thoracic portion of the respiratory system because Systemic it is enclosed in the thorax. circulation Bones and Muscles of the Thorax CO O2 Exchange III: 2 blood to cells Surround the Lungs CO 2 O2 Th e thorax is bounded by the bones of the spine and rib cage Cellular and their associated muscles. Together the bones and muscles Cells respiration Nutrients are called the thoracic cage . Th e ribs and spine (the chest wall ) ATP form the sides and top of the cage. A dome-shaped sheet of skel- etal muscle, the diaphragm , forms the fl oor (Fig. 17.2 b). Two sets of intercostal muscles , internal and external, Fig. 17.1 connect the 12 pairs of ribs (Fig. 17.2 a). Additional muscles, the sternocleidomastoids and the scalenes , run from the head and neck to the sternum and fi rst two ribs. dioxide, water, and energy in the form of ATP. External respi- Functionally, the thorax is a sealed container fi lled with three ration, the topic of this chapter and the next, is the movement membranous bags, or sacs. One, the pericardial sac, contains the of gases between the environment and the body’s cells. External heart. Th e other two bags, the pleural sacs, each surround a lung respiration can be subdivided into four integrated processes, il- {pleura, rib or side}. The esophagus and thoracic blood vessels lustrated in Figure 17.1 : and nerves pass between the pleural sacs ( Fig. 17.2 c). 1 Th e exchange of air between the atmosphere and the lungs. Th is process is known as ventilation , or breathing. Inspi- Pleural Sacs Enclose the Lungs ration () is the movement of air into the lungs. Th e lungs ( Fig. 17.2 b, d) consist of light, spongy tissue whose Expiration () is the movement of air out of the volume is occupied mostly by air-fi lled spaces. Th ese irregular lungs. Th e mechanisms by which ventilation takes place cone-shaped organs nearly fill the thoracic cavity, with their are collectively called the mechanics of breathing . bases resting on the curved diaphragm. Semi-rigid conducting 2 The exchange of O a n d C O between the lungs and the 2 2 airways—the bronchi—connect the lungs to the main airway, blood. the trachea. 3 Th e transport of O a n d C O by the blood. 2 2 Each lung is surrounded by a double-walled pleu- 4 Th e exchange of gases between blood and the cells. ral sac whose membranes line the inside of the thorax and External respiration requires coordination between the re- cover the outer surface of the lungs ( Fig. 17.3 ). Each pleu- spiratory and cardiovascular systems. Th e respiratory system ral membrane, or pleura , contains several layers of elastic

602 Mechanics of Breathing

connective tissue and numerous capillaries. Th e opposing layers Concept Check Answers: End of Chapter of pleural membrane are held together by a thin fi lm of pleu- ral fl uid whose total volume is only about 25–30 mL in a 70-kg 1. What is the diff erence between cellular respiration and external man. Th e result is similar to an air-fi lled balloon (the lung) sur- respiration? rounded by a water-fi lled balloon (the pleural sac). Most illus- 2. Name the components of the upper respiratory tract and those of the trations exaggerate the volume of the pleural fl uid, but you can lower respiratory tract. appreciate its thinness if you imagine spreading 25 mL of water 3. Based on the total cross-sectional area of diff erent airways, where is evenly over the surface of a 3-liter soft drink bottle. the velocity of air fl ow highest and lowest? Pleural fluid serves several purposes. First, it creates a moist, slippery surface so that the opposing membranes can 4. Give two functions of pleural fl uid. slide across one another as the lungs move within the thorax. 5. Name the components (including muscles) of the thoracic cage. List the Second, it holds the lungs tight against the thoracic wall. To vi- contents of the thorax. sualize this arrangement, think of two panes of glass stuck to- 6. Which air passages of the respiratory system are collapsible? gether by a thin fi lm of water. You can slide the panes back and forth across each other, but you cannot pull them apart because of the cohesiveness of the water. A similar fl uid bond between the two pleural membranes makes the lungs “stick” to the tho- The Airways Warm, Humidify, 17 racic cage and holds them stretched in a partially infl ated state, even at rest. and Filter Inspired Air During breathing, the upper airways and the bronchi do more than simply serve as passageways for air. Th ey play an important Airways Connect Lungs to the role in conditioning air before it reaches the alveoli. Condition- External Environment ing has three components: 1 Warming air to body temperature (37 ЊC), so that core Air enters the upper respiratory tract through the mouth and body temperature does not change and alveoli are not nose and passes into the pharynx , a common passageway damaged by cold air; for food, liquids, and air {pharynx, throat}. From the pharynx, 2 Adding water vapor until the air reaches 100% humidity, so air flows through the larynx into the trachea , or windpipe that the moist exchange epithelium does not dry out; and (Fig. 17.2 b). The larynx contains the vocal cords , connective 3 Filtering out foreign material, so that viruses, bacteria, and tissue bands that vibrate and tighten to create sound when air inorganic particles do not reach the alveoli. moves past them. The trachea is a semiflexible tube held open by 15 to 20 Inhaled air is warmed by the body’s heat and moistened by C-shaped cartilage rings. It extends down into the thorax, where water evaporating from the mucosal lining of the airways. Un- it branches (division 1) into a pair of primary bronchi, one der normal circumstances, by the time air reaches the trachea, bronchus to each lung (Fig. 17.2 b). Within the lungs, the bronchi it has been conditioned to 100% humidity and 37 ЊC . Breath- branch repeatedly (divisions 2–11) into progressively smaller ing through the mouth is not nearly as eff ective at warming and bronchi (Fig. 17.2 e). Like the trachea, the bronchi are semi-rigid moistening air as breathing through the nose. If you exercise tubes supported by cartilage. outdoors in very cold weather, you may be familiar with the Within the lungs, the smallest bronchi branch to become ache in your chest that results from breathing cold air through bronchioles, small collapsible passageways with walls of smooth your mouth. muscle. Th e bronchioles continue branching (divisions 12–23) Air is filtered both in the trachea and in the bronchi. until the respiratory bronchioles form a transition between the These airways are lined with ciliated epithelium whose cilia airways and the exchange epithelium of the lung. are bathed in a watery saline layer ( Fig. 17.5 ). The saline is The diameter of the airways becomes progressively produced by epithelial cells when Cl- secreted into the lumen smaller from the trachea to the bronchioles, but as the individ- by apical anion channels draws Na + into the lumen through the ual airways get narrower, their numbers increase geometrically paracellular pathway ( Fig. 17.5 c). Movement of solute from the ( Fig. 17.4 ). As a result, the total cross-sectional area in- ECF to the lumen creates an osmotic gradient, and water follows creases with each division of the airways. Total cross- the ions into the airways. Th e CFTR channel, whose malfunc- sectional area is lowest in the upper respiratory tract and tion causes cystic fi brosis, is one of the anion channels found on greatest in the bronchioles, analogous to the increase in cross- the apical surface of this epithelium. sectional area that occurs from the aorta to the capillaries in A sticky layer of mucus fl oats over the cilia to trap most the circulatory system. inhaled particles larger than 2 mm . Th e mucus layer is secreted

603 Fig. 17.2 ANATOMY SUMMARY

The Lungs and Thoracic Cavity

(a) Muscles of the thorax, neck, and abdomen (b) The respiratory system is divided create the force to move air during breathing. into upper and lower regions.

Pharynx

Nasal cavity Upper Vocal cords Sternocleido- respiratory mastoids Tongue system Scalenes Esophagus Larynx

Trachea

Lower respiratory system External Internal intercostals intercostals

Diaphragm Abdominal muscles

Left lung Right lung Muscles Muscles Diaphragm of inspiration of expiration Right bronchus Left bronchus

(c) Sectional view of chest. Each lung is enclosed in (d) On external view, the right lung is divided two pleural membranes. The esophagus and aorta into three lobes, and the left lung is pass through the thorax between the pleural sacs. divided into two lobes.

Apex Pleural Esophagus Aorta membranes

Superior lobe Superior lobe Right Left lung lung Middle lobe

Heart

Inferior Inferior lobe lobe

Right pleural Pericardial Left cavity cavity Base Cardiac notch

Superior view

604 The Bronchi and Alveoli

(e) Branching of airways creates (f) Structure of lung lobule. Each cluster of alveoli is about 80 million bronchioles. surrounded by elastic fibers and a network of capillaries.

Larynx Bronchiole Branch of pulmonary artery

Bronchial artery, Smooth muscle nerve and vein The trachea branches into Trachea two primary bronchi. Branch of Elastic Left primary pulmonary fibers Cartilage bronchus vein ring Capillary Lymphatic beds vessel The primary bronchus divides 22 more times, terminating in a cluster Secondary of alveoli. bronchus

Alveoli

Bronchiole

Alveoli

(g) Alveolar structure

(h) Exchange surface of alveoli Capillary Elastic fibers

Alveolar Nucleus of epithelium endothelial cell

RBC Type I alveolar cell for gas exchange Capillary Endothelium Plasma Endothelial cell of capillary 0.1- 1.5 Type II alveolar μm cell (surfactant cell) synthesizes Alveolar surfactant. air space Alveolus Surfactant Fused Limited basement interstitial membranes fluid

Alveolar Blue arrow represents gas exchange macrophage between alveolar air space and the plasma. ingests foreign material.

605 Mechanics of Breathing

THE PLEURAL SAC Concept Check Answers: End of Chapter

The pleural sac forms a double membrane surrounding the lung, 7. Cigarette smoking paralyzes cilia in the airways and increases mucus similar to a fluid-filled balloon surrounding an air-filled balloon. production. Why would these eff ects cause smokers to develop a cough? Pleural membrane Air-filled Air space balloon of lung Alveoli Are the Site of Gas Exchange Fluid-filled balloon The pleural fluid has a much Th e alveoli, clustered at the ends of terminal bronchioles, make smaller volume than is up the bulk of lung tissue (Fig. 17.2 f, g). Th eir primary function suggested by this illustration. is the exchange of gases between themselves and the blood. Fig. 17.3 Each tiny alveolus is composed of a single layer of epithelium ( Fig. 17.2 g). Two types of epithelial cells are found in the alveoli. The smaller but thicker type II alveolar cells by goblet cells in the epithelium ( Fig. 17.5 b). Th e cilia beat with synthesize and secrete a chemical known as surfactant. an upward motion that moves the mucus continuously toward Surfactant mixes with the thin fl uid lining of the alveoli to aid the pharynx, creating what is called the mucociliary escalator . lungs as they expand during breathing, as you will see later in Mucus contains immunoglobulins that can disable many patho- this chapter. Type II cells also help minimize the amount of fl uid gens. Once mucus reaches the pharynx, it can be spit out (expec- present in the alveoli by transporting solutes, followed by water, torated ) or swallowed. For swallowed mucus, stomach acid and out of the alveolar air space. enzymes destroy any remaining microorganisms. The larger type I alveolar cells occupy about 95% of Secretion of the watery saline layer beneath the mucus is the alveolar surface area and are very thin so that gases can dif- essential for a functional mucociliary escalator. In the disease fuse rapidly through them (Fig. 17.2 h). In much of the exchange cystic fi brosis, for example, inadequate ion secretion decreases area, a layer of basement membrane fuses the alveolar epithe- fl uid movement in the airways. Without the saline layer, cilia be- lium to the capillary endothelium. In the remaining area only a come trapped in thick, sticky mucus. Mucus cannot be cleared, small amount of interstitial fl uid is present. and bacteria colonize the airways, resulting in recurrent lung The thin walls of the alveoli do not contain muscle be- infections. cause muscle fi bers would block rapid gas exchange. As a result,

BRANCHING OF THE AIRWAYS Cross-sectional Name Division Diameter (mm) How many? area (cm2)

Conducting system Trachea 015-22 1 2.5

Primary bronchi 1210-15

Smaller 2 4 bronchi 3

4 1-10

5

6-11 1 x 104

2 x 104 100 Bronchioles 12-23 0.5-1 Exchange surface 8 x 107 5 x 103

Alveoli 24 0.3 3-6 x 108 >1 x 106

Fig. 17.4

606 Mechanics of Breathing

AIRWAY EPITHELIUM

(a) Epithelial cells lining the airways and submucosal (b) Cilia move the mucus layer toward the pharynx, removing trapped glands secrete saline and mucus. pathogens and particulate matter.

Dust particle

Ciliated Mucus layer traps epithelium inhaled particles.

Watery saline layer allows cilia to push mucus toward pharynx.

Cilia

Goblet cell secretes mucus.

Movement of mucus Nucleus of columnar 17 Mucus layer Submucosal epithelial cell gland Lumen of airway Basement membrane

(c) One model of saline secretion by airway epithelial cells

Saline layer Na+ H O – in lumen 2 Cl

2 1 NKCC brings Cl– into epithelial cell from ECF. Anion Respiratory channel epithelial 2 Apical anion channels, cells including CFTR, allow Cl– to enter the lumen.

3 Na+ goes from ECF to lumen by the paracelllular pathway, drawn by the electrochemical gradient. K+

ATP 4 NaCl movement from ECF to + 3 Na + + – + lumen creates a concentration Na Na 2Cl K K+ H2O gradient so water follows into ECF 1 the lumen. 4

Fig. 17.5 lung tissue itself cannot contract. However, connective tis- proximity of capillary blood to alveolar air is essential for the sue between the alveolar epithelial cells contains many elastin rapid exchange of gases. and collagen fi bers that create when lung tissue is stretched. Pulmonary Circulation Th e close association of the alveoli with an extensive net- work of capillaries demonstrates the intimate link between the Is High-Flow, Low-Pressure respiratory and cardiovascular systems. Blood vessels fi ll 80– Th e pulmonary circulation begins with the pulmonary trunk, 90% of the space between alveoli, forming an almost continuous which receives low-oxygen blood from the right ventricle. Th e “sheet” of blood in close contact with the air-fi lled alveoli. Th e pulmonary trunk divides into two pulmonary arteries, one to

607 Mechanics of Breathing

CLINICAL FOCUS Concept Check Answers: End of Chapter 8. Is blood fl ow through the pulmonary trunk greater than, less than, or Congestive Heart Failure equal to blood fl ow through the aorta? When is a lung problem not a lung problem? The 9. A person has left ventricular failure but normal right ventricular answer: when it’s really a heart problem. Congestive heart function. As a result, blood pools in the pulmonary circulation, failure (CHF) is an excellent example of the interrelationships doubling pulmonary capillary hydrostatic pressure. What happens to among body systems and demonstrates how disruptions net fl uid fl ow across the walls of the pulmonary capillaries? in one system can have a domino eff ect in the others. The 10. Calculate the mean pressure in a person whose pulmonary arterial primary symptoms of heart failure are shortness of breath pressure is 25 8 mm Hg. ( dyspnea ), wheezing during breathing, and sometimes a > productive cough that may be pinkish from the presence of blood. Congestive heart failure arises when the right heart is a more eff ective pump than the left heart. When blood accumulates in the pulmonary circulation, increased Gas Laws volume increases pulmonary blood pressure and capillary hydrostatic pressure. Capillary fi ltration exceeds the ability Respiratory air fl ow is very similar in many respects to blood of the lymph system to drain interstitial fl uid, resulting in fl ow in the cardiovascular system because both air and blood pulmonary edema. Treatment of CHF includes increasing are fluids. Their primary difference is that blood is a non- urinary output, which brings yet another organ system into compressible liquid but air is a compressible mixture of gases. the picture. By current estimates, about 5 million Americans Figure 17.6 summarizes the laws that govern the behavior of suff er from CHF. To learn more about this condition, visit the gases in air and provide the basis for the exchange of air between American Heart Association web site ( www.americanheart. the external environment and the alveoli. org ) or MedlinePlus, published by the National Institutes of In this course, blood pressure and environmental air pres- Health ( www.nlm.nih.gov/medlineplus/heartfailure.html ). sure ( ) are both reported in millimeters of mercury (mm Hg). Respiratory physiologists sometimes report gas pressures in centimeters of water instead, where 1 mm Hg = 1.36 cm H2O , or in kiloPascals (kPa), where 760 mm Hg = each lung. Oxygenated blood from the lungs returns to the left 101.325 kPa. atrium via the pulmonary veins. At sea level, normal atmospheric pressure is 760 mm Hg. At any given moment, the pulmonary circulation contains However, in this course we follow the convention of designating about 0.5 liter of blood, or 10% of total blood volume. About 75 mL atmospheric pressure as 0 mm Hg. Because atmospheric pres- of this amount is found in the capillaries, where gas exchange sure varies with altitude and because very few people live ex- takes place, with the remainder in pulmonary arteries and veins. actly at sea level, this convention allows us to compare pressure Th e rate of blood fl ow through the lungs is much higher than the diff erences that occur during ventilation without correcting for rate in other tissues because the lungs receive the entire cardiac altitude. output of the right ventricle: 5 L min. Th is means that as much > blood fl ows through the lungs in one minute as fl ows through the entire rest of the body in the same amount of time! RUNNING PROBLEM Despite the high fl ow rate, pulmonary blood pressure is low. Edna has not been able to stop smoking, and her COPD Pulmonary arterial pressure averages 25 8 mm Hg, much lower > is a combination of emphysema and bronchitis. Patients than the average systemic pressure of 120 80 mm Hg. Th e right > with chronic bronchitis have excessive mucus production ventricle does not have to pump as forcefully to create blood fl ow and exhibit general infl ammation of the entire respiratory through the lungs because resistance of the pulmonary circulation tract. The mucus narrows the airways and makes breathing is low. Th is low resistance can be attributed to the shorter total diffi cult. length of pulmonary blood vessels and to the distensibility and large total cross-sectional area of pulmonary arterioles. Q1: What does narrowing of the airways do to airway Normally, the net hydrostatic pressure fi ltering fl uid out of resistance? a pulmonary capillary into the interstitial space is low because of low mean blood pressure. Th e lymphatic system effi ciently re- moves fi ltered fl uid, and lung interstitial fl uid volume is usually minimal. As a result, the distance between the alveolar air space and the capillary endothelium is short, and gases diff use rapidly between them.

608 Fig. 17.6 ESSENTIALS

Gas Laws

This figure summarizes the rules that govern the behavior of gases in air. These rules provide the basis for the exchange of air between the external environment and the alveoli.

(a) The ideal gas equation

Where P is pressure, V is volume, n is the moles of gas, T is absolute PV = nRT temperature, and R is the universal gas constant, 8.3145 j/mol × K

In the human body we can assume that the number of moles and temperature are constant. Removing the constants leaves the following equation:

This relationship says that if the volume of gas V = 1/P increases, the pressure decreases, and vice versa.

(b) Boyle’s Law

Boyle’s law also expresses this inverse relationship between pressure and volume.

For example, the container on the left is 1 L (V1) P1V1 = P2V2 and has a pressure of 100 mm Hg (P1).

What happens to the pressure when the volume decreases to 0.5 L?

100 mm Hg × 1 L = P2 × 0.5 L

200 mm Hg = P2 The pressure has increased ×2.

The Ideal Gas law and Boyle’s law apply V = 1.0 L V = 0.5 L to all gases or mixtures of gases. 1 2 P1 = 100 mm Hg P2 = 200 mm Hg

(c) Dalton’s Law

Dalton’s law says that the total pressure of a mixture of gases is the sum of the pressures of the individual gases. The pressure of an individual gas in a mixture is known as the partial pressure of the gas (Pgas).

For example, at sea level, atmospheric pressure (Patm) is 760 mm Hg, In humid air, water vapor “dilutes” the contribution and oxygen is 21% of the atmosphere. What is the partial pressure of of other gases to the total pressure. oxygen (P )? O2 Partial Pressures (P ) of Atmospheric Gases at 760 mm Hg To find the partial pressure of any one gas in a sample gas of dry air, multiply the atmospheric pressure (Patm) by the gas’s Gas and its P in dry P in P in relative contribution (%) to P : gas gas gas atm percentage in air 25 ˚C air 25 ˚C air, 37 ˚C air, 100% humidity 100% humidity Partial pressure of a gas = P × % of gas in atmosphere atm Oxygen (O2) 21% 160 mm Hg 155 mm Hg 150 mm Hg

Carbon dioxide P = 760 mm Hg x 21% P 0.25 mm Hg 0.24 mm Hg 0.235 mm Hg O2 O2 (CO2) 0.03%

= 760 mm × 0.21 = 160 mm Hg Water vapor 0 mm Hg 24 mm Hg 47 mm Hg The partial pressure of oxygen (P ) in dry air at sea level O2 is 160 mm Hg. To calculate the partial pressure of a gas in humid air, you must first subtract the water vapor pressure from the total pressure. At 100% humidity and 25° C, water vapor pressure (P ) is 24 mm Hg. The pressure exerted by an individual gas is determined only by its H2O relative abundance in the mixture and is independent of the molecular size or mass of the gas. P in humid air = (P – P ) × % of gas gas atm H2O

P = (760 – 24) × 21% = 155 mm Hg O2

609 Mechanics of Breathing

Air Is a Mixture of Gases Boyle’s Law Describes Th e atmosphere surrounding the earth is a mixture of gases and Pressure-Volume Relationships water vapor. Dalton’s law states that the total pressure exerted Th e pressure exerted by a gas or mixture of gases in a sealed con- by a mixture of gases is the sum of the pressures exerted by the tainer is created by the collisions of moving gas molecules with individual gases ( Fig. 17.6 c). For example, in dry air at an atmo- the walls of the container and with each other. If the size of the spheric pressure of 760 mm Hg, 78% of the total pressure is due container is reduced, the collisions between the gas molecules to N2 , 21% to O2 , and so on. and the walls become more frequent, and the pressure rises In respiratory physiology, we are concerned not only with (Fig. 17.6 b). Th is relationship between pressure and volume was total atmospheric pressure but also with the individual pres- fi rst noted by Robert Boyle in the 1600s and can be expressed by sures of oxygen and carbon dioxide. Th e pressure of a single gas the equation of Boyle’s law of gases: in a mixture is known as its partial pressure ( Pgas ). Th e pressure exerted by an individual gas is determined only by its relative = P 1V1 P2V2 abundance in the mixture and is independent of the molecular size or mass of the gas. where P represents pressure and V represents volume. Th e partial pressures of gases in air vary slightly depending Boyle’s law states that if the volume of a gas is reduced, the on how much water vapor is in the air because the pressure of pressure increases. If the volume increases, the pressure decreases. water vapor “dilutes” the contribution of other gases to the total In the respiratory system, changes in the volume of the pressure. Th e table in Figure 17.6 c compares the partial pres- chest cavity during ventilation cause pressure gradients that sures of some gases in dry air and at 100% humidity. create air fl ow. When chest volume increases, alveolar pressure falls, and air fl ows into the respiratory system. When the chest volume decreases, alveolar pressure increases, and air fl ows out into the atmosphere. Th is movement of air is bulk fl ow because Concept Check Answers: End of Chapter the entire gas mixture is moving rather than merely one or two 11. If nitrogen is 78% of atmospheric air, what is the partial pressure of of the gases in the air. nitrogen (P ) in a sample of dry air that has an atmospheric pressure N2 of 720 mm Hg? 12. The partial pressure of water vapor in inspired air is 47 mm Hg when Ventilation inhaled air is fully humidifi ed. If atmospheric pressure is 700 mm Hg Th is bulk fl ow exchange of air between the atmosphere and the and oxygen is 21% of the atmosphere at 0% humidity, what is the P O2 alveoli is ventilation, or breathing ( Fig. 17.1 ). A single respira- of fully humidifi ed air? tory cycle consists of an inspiration followed by an expiration.

Lung Volumes Change During Ventilation Physiologists and clinicians assess a person’s pulmonary func- tion by measuring how much air the person moves during quiet Gases Move Down Pressure Gradients breathing, then with maximum eff ort. Th ese pulmonary func- Air fl ow occurs whenever there is a pressure gradient. Bulk fl ow tion tests use a spirometer , an instrument that measures the of air, like blood fl ow, is directed from areas of higher pressure volume of air moved with each breath ( Fig. 17.7 a). (Most spi- to areas of lower pressure. Meteorologists predict the weather by rometers in clinical use today are small computerized machines knowing that areas of high atmospheric pressure move in to re- rather than the traditional wet spirometer illustrated here.) place areas of low pressure. In ventilation, bulk fl ow of air down When a subject is attached to the traditional spirometer pressure gradients explains how air is exchanged between the through a mouthpiece and the subject’s nose is clipped closed, external environment and the lungs. Movement of the thorax the subject’s respiratory tract and the spirometer form a closed during breathing creates alternating conditions of high and low system. When the subject breathes in, air moves from the spi- pressure in the lungs. rometer into the lungs, and the recording pen, which traces a Diff usion of gases down concentration (partial pressure) graph on a rotating cylinder, moves up. When the subject ex- gradients applies to single gases. For example, oxygen moves hales, air moves from the lungs back into the spirometer, and from areas of higher oxygen partial pressure (P ) to areas of the pen moves down. O2 lower oxygen partial pressure. Diff usion of individual gases is important in the exchange of oxygen and carbon dioxide be- Lung Volumes Th e air moved during breathing can be divided tween alveoli and blood and from blood to cells. into four lung volumes: (1) , (2) inspiratory reserve

610 Mechanics of Breathing

PULMONARY FUNCTION TESTS

(a) The Spirometer

This figure shows a traditional wet spirometer. The subject inserts a mouthpiece that is attached to an inverted bell filled with air or oxygen. The volume of the bell and the volume of the subject’s respiratory tract create a closed system because the bell is suspended in water. Bell

Inspiration Expiration Inspiration Expiration

Air 0.5 0.5

Volume (L)

Water 0 Time 17 When the subject inhales, air moves into the lungs. The volume of the bell decreases, and the pen rises on the tracing.

(b) Lung Volumes and Capacities

The four lung volumes A spirometer tracing showing lung volumes and capacities.

Dead space 5800 RV

ERV Inspiratory VT Inspiratory capacity KEY reserve IRV RV = Residual volume volume End of normal ERV = Expiratory reserve volume 3000 mL inspiration VT = Tidal volume IRV = Inspiratory reserve volume Tidal Vital volume capacity 4600 mL 2800 500mL Total Capacities are sums of 2 or more volumes. lung 2300 capacity Inspiratory capacity = VT + IRV Expiratory = VT + IRV + ERV End of normal reserve Total lung capacity = VT + IRV + ERV + RV Volume expiration Functional residual capacity = ERV + RV (mL) volume 1100 mL Functional 1200 residual Pulmonary Volumes and Capacities* capacity Residual Males Females volume 1200 mL IRV 3000 1900 Vital V 500 500 capacity T Time ERV 1100 700 Residual volume 1200 1100

Total lung 5800 mL 4200 mL capacity *Pulmonary volumes are given for a normal 70-kg man or a 50-kg woman, 28 years old. Fig. 17.7

611 Mechanics of Breathing

volume, (3) expiratory reserve volume, and (4) residual volume. During Ventilation, Air Flows Because Th e numerical values used on the graph in Figure 17.7 b represent average volumes for a 70-kg man. Th e volumes for women are typ- of Pressure Gradients ically less, as shown in Figure 17.7 b. Lung volumes vary consider- Breathing is an active process that requires muscle contraction. ably with age, sex, height, and weight, so clinicians use algorithms Air fl ows into the lungs because of pressure gradients created based on those parameters to predict lung volumes. (An algorithm by a pump, just as blood fl ows because of the pumping action is an equation or series of steps used to solve a problem.) of the heart. In the respiratory system, muscles of the thoracic Each of the following paragraphs begins with the instructions cage and diaphragm function as the pump because most lung you would be given if you were being tested for these volumes. tissue is thin exchange epithelium. When these muscles con- “Breathe quietly .” Th e volume of air that moves during a sin- tract, the lungs expand, held to the inside of the chest wall by the gle inspiration or expiration is known as the tidal volume ( VT ) . pleural fl uid. Average tidal volume during quiet breathing is about 500 mL. (It is Th e primary muscles involved in quiet breathing (breath- hard for subjects to breathe normally when they are thinking about ing at rest) are the diaphragm, the external intercostals, and the their breathing, so the clinician may not give this instruction.) scalenes. During forced breathing, other muscles of the chest “ Now, at the end of a quiet inspiration, take in as much addi- and abdomen may be recruited to assist. Examples of physio- tional air as you possibly can. ” Th e additional volume you inspire logical situations in which breathing is forced include exercise, above the tidal volume represents your inspiratory reserve vol- playing a wind instrument, and blowing up a balloon. ume (IRV). In a 70-kg man, this volume is about 3000 mL, a As we noted earlier in the chapter, air fl ow in the respira- sixfold increase over the normal tidal volume. tory tract obeys the same rule as blood fl ow: “Now stop at the end of a normal exhalation, then exhale Flow r ⌬ P R as much air as you possibly can. ” Th e amount of air forcefully > exhaled aft er the end of a normal expiration is the expiratory reserve volume (ERV), which averages about 1100 mL. This equation means that (1) air flows in response to a ⌬ The fourth volume cannot be measured directly. Even pressure gradient ( P ) and (2) fl ow decreases as the resistance if you blow out as much air as you can, air still remains in the (R) of the system to fl ow increases. Before we discuss resistance, lungs and the airways. Th e volume of air in the respiratory sys- let’s consider how the respiratory system creates a pressure gra- tem aft er maximal exhalation—about 1200 mL—is called the re- dient. Th e pressure-volume relationships of Boyle’s law provide sidual volume (RV). Most of this residual volume exists because the basis for pulmonary ventilation. the lungs are held stretched against the ribs by the pleural fl uid.

Lung Capacities Th e sum of two or more lung volumes is called Concept Check Answers: End of Chapter a capacity. Th e vital capacity (VC) is the sum of the inspiratory re- 17. Compare the direction of air movement during one respiratory cycle serve volume, expiratory reserve volume, and tidal volume. Vital with the direction of blood fl ow during one cardiac cycle. capacity represents the maximum amount of air that can be volun- tarily moved into or out of the respiratory system with one breath. 18. Explain the relationship between the lungs, the pleural membranes, the pleural fl uid, and the thoracic cage. To measure vital capacity, you would instruct the person to take in as much air as possible, then blow it all out. Vital capacity decreases with age as muscles weaken and the lungs become less elastic. Vital capacity plus the residual volume yields the total lung capacity (TLC). Other capacities of importance in pulmo- Inspiration Occurs When nary medicine include the inspiratory capacity (tidal volume + Alveolar Pressure Decreases inspiratory reserve volume) and the functional residual capac- For air to move into the alveoli, pressure inside the lungs must ity (expiratory reserve volume + residual volume). become lower than atmospheric pressure. According to Boyle’s law, an increase in volume will create a decrease in pressure. Concept Check Answers: End of Chapter During inspiration, thoracic volume increases when certain 13. How are lung volumes related to lung capacities? skeletal muscles of the rib cage and diaphragm contract. When the diaphragm contracts, it drops down toward 14. Which lung volume cannot be measured directly? the abdomen. In quiet breathing, the diaphragm moves about 15. If vital capacity decreases with age but total lung capacity does not 1.5 cm, increasing thoracic volume ( Fig. 17.8 b). Contraction change, which lung volume must be changing? In which direction? of the diaphragm causes between 60% and 75% of the inspira- 16. As inhaled air becomes humidifi ed passing down the airways, what tory volume change during normal quiet breathing. happens to the P of the air? Movement of the rib cage creates the remaining 25–40% of O2 the volume change. During inhalation, the external intercostal

612 Mechanics of Breathing

MOVEMENT OF THE THORACIC CAGE AND DIAPHRAGM DURING BREATHING

(a) At rest: Diaphragm is relaxed.

Pleural space During inspiration, the dimensions of the thoracic cavity increase.

Vertebrae Sternum Rib

Diaphragm

(b) Inspiration: Thoracic volume increases.

17

Side view: “Pump handle" motion increases anterior-posterior dimension of rib cage. Movement of the handle on a hand pump is analogous to the lifting of the sternum and ribs.

Vertebrae Rib Diaphragm contracts and flattens.

(c) Expiration: Diaphragm relaxes, thoracic volume decreases.

Sternum Front view: “Bucket handle" motion increases lateral dimension of rib cage. The bucket handle moving up and out is a good model for lateral rib movement during inspiration.

Fig. 17.8

and scalene muscles (see Fig. 17.2 a) contract and pull the ribs however, studies have changed our understanding of how these upward and out ( Fig. 17.8 b). Rib movement during inspiration accessory muscles contribute to quiet breathing. has been likened to a pump handle lift ing up and away from the If an individual’s scalenes are paralyzed, inspiration is pump (the ribs moving up and away from the spine) and to the achieved primarily by contraction of the diaphragm. Observa- movement of a bucket handle as it lift s away from the side of a tion of patients with neuromuscular disorders has revealed that bucket (ribs moving outward in a lateral direction). Th e combi- although the contracting diaphragm increases thoracic volume nation of these two movements broadens the rib cage in all di- by moving toward the abdominal cavity, it also tends to pull the rections. As thoracic volume increases, pressure decreases, and lower ribs inward, working against inspiration. In normal indi- air fl ows into the lungs. viduals, we know that the lower ribs move up and out during For many years, quiet breathing was attributed solely to inspiration rather than inward. Th e fact that there is no up-and- the action of the diaphragm and the external intercostal mus- out rib motion in patients with paralyzed scalenes tells us that cles. It was thought that the scalenes and sternocleidomastoid normally the scalenes must be contributing to inspiration by muscles were active only during deep breathing. In recent years, lift ing the sternum and upper ribs.

613 Mechanics of Breathing

New evidence also downplays the role of the external in- RUNNING PROBLEM tercostal muscles during quiet breathing. However, the external Edna’s COPD began with chronic bronchitis and a morning intercostals play an increasingly important role as respiratory cough that produced lots of mucus (phlegm ). Cigarette activity increases. Because the exact contribution of external in- smoke paralyzes the cilia that sweep debris and mucus out of tercostals and scalenes varies depending on the type of breath- the airways, and smoke irritation increases mucus production ing, we group these muscles together and simply call them the in the airway. Without functional cilia, mucus and debris inspiratory muscles . pool in the airways, leading to a chronic cough. Eventually, Now let’s see how alveolar pressure ( PA) changes during smokers may begin to develop emphysema in addition to a single inspiration. Follow the graphs in Figure 17.9 as you their bronchitis. read through the process. Remember that atmospheric pressure Q2: Why do people with chronic bronchitis have a higher- is assigned a value of 0 mm Hg. Negative numbers designate than-normal rate of respiratory infections? subatmospheric pressures, and positive numbers denote higher- than-atmospheric pressures. Time 0. In the brief pause between breaths, alveolar pres- sure is equal to atmospheric pressure (0 mm Hg at point A1 ) . When pressures are equal, there is no air fl ow.

PRESSURE CHANGES DURING QUIET BREATHING

Inspiration Expiration Inspiration Expiration +2 Alveolar A4 pressure • • +1 Trachea (mm Hg)

A1• • • • •0 • A3 A5 Bronchi • • –1 A2 –2 • Intrapleural Lung B3 B pressure • 1• ••–3 (mm Hg)

–4

–5 Diaphragm • • –6 B2 Right pleural Left pleural cavity cavity 750 Volume of air moved GRAPH QUESTIONS (mL) • • 500 C2 1. At what point in the cycle is alveolar pressure greatest? Least? Equal to atmospheric pressure? 2. When lung volume is at its minimum, alveolar pressure 250 is ______and external intercostal muscle contraction is______.

C3 (a) maximun C1• • • (b) minimum 01 2 3 4 5678 (c) moving from maximum to minimum Time (sec) (d) moving from minimum to maximum Normally expiration takes 2–3 times longer than inspiration 3. What is this person’s ventilation rate? (not shown to scale on this idealized graph). Fig. 17.9

614 Mechanics of Breathing

Time 0–2 sec: Inspiration. As inspiration begins, inspira- The internal intercostal muscles line the inside of the rib tory muscles contract, and thoracic volume increases. With the cage. When they contract, they pull the ribs inward, reducing the increase in volume, alveolar pressure falls about 1 mm Hg below volume of the thoracic cavity. To feel this action, place your hands - atmospheric pressure ( 1 mm Hg, point A 2), and air fl ows into on your rib cage. Forcefully blow as much air out of your lungs as the alveoli (point C1 t o p o i n t C 2 ). Because the thoracic volume you can, noting the movement of your hands as you do so. changes faster than air can flow, alveolar pressure reaches its Th e internal and external intercostals function as antago- lowest value about halfway through inspiration (point A2 ) . nistic muscle groups to alter the position and volume of the rib As air continues to fl ow into the alveoli, pressure increases cage during ventilation. The diaphragm, however, has no an- until the thoracic cage stops expanding, just before the end of tagonistic muscles. Instead, abdominal muscles contract dur- inspiration. Air movement continues for a fraction of a second ing active expiration to supplement the activity of the internal longer, until pressure inside the lungs equalizes with atmo- intercostals. Abdominal contraction pulls the lower rib cage spheric pressure (point A 3). At the end of inspiration, lung vol- inward and decreases abdominal volume, actions that displace ume is at its maximum for the respiratory cycle (point C 2 ) , a n d the intestines and liver upward. The displaced viscera push alveolar pressure is equal to atmospheric pressure. the diaphragm up into the thoracic cavity and passively decrease You can demonstrate this phenomenon by taking a deep chest volume even more. Th e action of abdominal muscles dur- breath and stopping the movement of your chest at the end of ing forced expiration is why aerobics instructors tell you to blow inspiration. (Do not “hold your breath” because doing so closes air out as you lift your head and shoulders during abdominal 17 the opening of the pharynx and prevents air fl ow.) If you do this “crunches.” Th e active process of blowing air out helps contract correctly, you notice that air fl ow stops aft er you freeze the in- the abdominals, the muscles you are trying to strengthen. spiratory movement. Th is exercise shows that at the end of in- Any neuromuscular disease that weakens skeletal muscles spiration, alveolar pressure is equal to atmospheric pressure. or damages their motor neurons can adversely aff ect ventilation. With decreased ventilation, less fresh air enters the lungs. In ad- dition, loss of the ability to cough increases the risk of pneu- Expiration Occurs When monia and other infections. Examples of diseases that aff ect the motor include myasthenia gravis, an ill- Alveolar Pressure Increases ness in which acetylcholine receptors of the motor end plates of At the end of inspiration, impulses from somatic motor neurons skeletal muscles are destroyed, and polio (poliomyelitis), a viral to the inspiratory muscles cease, and the muscles relax. Elastic illness that paralyzes skeletal muscles. recoil of the lungs and thoracic cage returns the diaphragm and rib cage to their original relaxed positions, just as a stretched elastic waistband recoils when released. Because expiration dur- Concept Check Answers: End of Chapter ing quiet breathing involves passive elastic recoil rather than ac- tive muscle contraction, it is called passive expiration. 19. Scarlett O’Hara is trying to squeeze herself into a corset with an 18-inch Time 2–4 sec: expiration. As lung and thoracic volumes waist. Will she be more successful by taking a deep breath and holding decrease during expiration, air pressure in the lungs increases, it or by blowing all the air out of her lungs? Why? reaching a maximum of about 1 mm Hg above atmospheric 20. Why would loss of the ability to cough increase the risk of respiratory pressure ( Fig. 17.9 , point A4). Alveolar pressure is now higher infections? (Hint : What does coughing do to mucus in the airways?) than atmospheric pressure, so air fl ow reverses and air moves out of the lungs. Time 4 sec. At the end of expiration, air movement ceases when alveolar pressure is again equal to atmospheric pressure Intrapleural Pressure Changes ( p o i n t A ). Lung volume reaches its minimum for the respira- 5 During Ventilation tory cycle (point C3 ). At this point, the respiratory cycle has ended and is ready to begin again with the next breath. Ventilation requires that the lungs, which are unable to expand The pressure differences shown in Figure 17.9 apply to and contract on their own, move in association with the expan- quiet breathing. During exercise or forced heavy breathing, sion and relaxation of the thorax. As we noted earlier in this these values become proportionately larger. Active expiration chapter, the lungs are enclosed in the fluid-filled pleural sac. occurs during voluntary and when ventilation ex- Th e surface of the lungs is covered by the visceral pleura , and ceeds 30–40 breaths per minute. (Normal resting ventilation the portion of the sac that lines the thoracic cavity is called the rate is 12–20 breaths per minute for an adult.) Active expiration parietal pleura { paries, wall}. Cohesive forces exerted by the fl uid uses the internal intercostal muscles and the abdominal muscles between the two pleural membranes cause the stretchable lung (see Fig. 17.2 a), which are not used during inspiration. Th ese to adhere to the thoracic cage. When the thoracic cage moves muscles are collectively called the expiratory muscles . during breathing, the lungs move with it.

615 Mechanics of Breathing

so the elastic lungs are forced to stretch to conform to the larger RUNNING PROBLEM volume of the thoracic cavity. At the same time, however, elastic Emphysema is characterized by a loss of elastin, the elastic recoil of the lungs creates an inwardly directed force that tries fi bers that help the alveoli recoil during expiration. Elastin to pull the lungs away from the chest wall ( Fig. 17.10 a). Th e is destroyed by elastase , an enzyme released by alveolar combination of the outward pull of the thoracic cage and inward macrophages, which must work overtime in smokers to rid recoil of the elastic lungs creates a subatmospheric intrapleural the lungs of irritants. People with emphysema have more pressure of about - 3 mm Hg. diffi culty exhaling than inhaling. Their alveoli have lost elastic You can create a similar situation by half-fi lling a syringe recoil, which makes expiration—normally a passive process— with water and capping it with a plugged-up needle. At this require conscious eff ort. point, the pressure inside the barrel is equal to atmospheric Q3: Name the muscles that patients with emphysema use to pressure. Now hold the syringe barrel (the chest wall) in one exhale actively. hand while you try to withdraw the plunger (the elastic lung pulling away from the chest wall). As you pull on the plunger, the volume inside the barrel increases slightly, but the cohe- sive forces between the water molecules cause the water to re- sist expansion. Th e pressure in the barrel, which was initially Th e intrapleural pressure in the fl uid between the pleural equal to atmospheric pressure, decreases slightly as you pull membranes is normally subatmospheric. Th is subatmospheric on the plunger. If you release the plunger, it snaps back to its pressure arises during fetal development, when the thoracic resting position, restoring atmospheric pressure inside the cage with its associated pleural membrane grows more rapidly syringe. than the lung with its associated pleural membrane. The two What happens to subatmospheric intrapleural pressure if pleural membranes are held together by the pleural fl uid bond, an opening is made between the sealed pleural cavity and the atmosphere? A knife thrust between the ribs, a broken rib that punctures the pleural membrane, or any other event that opens

SUBATMOSPHERIC PRESSURE IN THE PLEURAL CAVITY HELPS KEEP THE LUNGS INFLATED

(a) In the normal lung at rest, pleural fluid keeps the lung (b) . If the sealed pleural cavity is opened to the adhered to the chest wall. atmosphere, air flows in. The bond holding the lung to the chest wall is broken, and the lung collapses, creating a pneumothorax (air in the thorax).

Ribs P = -3 mm Hg P = Patm Intrapleural pressure is subatmospheric. Knife Lung collapses to Air unstretched size

Pleural fluid Pleural Visceral pleura membranes Parietal pleura

Diaphragm The rib cage expands slightly.

Elastic recoil of the Elastic recoil of lung If the sealed pleural cavity is opened chest wall tries to pull creates an inward pull. to the atmosphere, air flows in. the chest wall outward. Fig. 17.10

616 Mechanics of Breathing the pleural cavity to the atmosphere allows air to fl ow down its Concept Check Answers: End of Chapter pressure gradient into the cavity, just as air enters when you break the seal on a vacuum-packed can. 21. A person has periodic spastic contractions of the diaphragm, otherwise Air in the pleural cavity breaks the fl uid bond holding the known as hiccups. What happens to intrapleural and alveolar pressures lung to the chest wall. The chest wall expands outward while when a person hiccups? the elastic lung collapses to an unstretched state, like a defl ated 22. A stabbing victim is brought to the emergency room with a knife balloon (Fig. 17.10 b). This condition, called pneumothorax wound between the ribs on the left side of his chest. What has probably {pneuma, air + thorax, chest}, results in a collapsed lung that happened to his left lung? To his right lung? Why does the left side of is unable to function normally. Pneumothorax can also occur his rib cage seem larger than the right side? spontaneously if a congenital bleb, a weakened section of lung tissue, ruptures, allowing air from inside the lung to enter the pleural cavity. Correction of a pneumothorax has two components: re- Lung Compliance and Elastance moving as much air from the pleural cavity as possible with a suction pump, and sealing the hole to prevent more air from May Change in Disease States entering. Any air remaining in the cavity is gradually absorbed Adequate ventilation depends on the ability of the lungs to ex- into the blood, restoring the pleural fl uid bond and reinfl ating pand normally. Most of the work of breathing goes into over- 17 the lung. coming the resistance of the elastic lungs and the thoracic cage Pressures in the pleural fluid vary during a respiratory to stretching. Clinically, the ability of the lung to stretch is called cycle. At the beginning of inspiration, intrapleural pressure is compliance . about - 3 mm Hg (Fig. 17.9 , point B1). As inspiration proceeds, Compliance refers to the amount of force that must be ex- the pleural membranes and lungs follow the expanding thoracic erted in a body to deform it. In the lung, we can express compli- cage because of the pleural fl uid bond, but the elastic lung tissue ance as the change of volume (V) that results from a given force resists being stretched. Th e lungs attempt to pull farther away or pressure (P) exerted on the lung: ⌬ V ⌬ P. A high-compliance > from the chest wall, causing the intrapleural pressure to become lung stretches easily, just as a compliant person is easy to per- even more negative ( Fig. 17.9 , point B2 ) . suade. A low-compliance lung requires more force from the in- Because this process is diffi cult to visualize, let’s return to spiratory muscles to stretch it. the analogy of the water-fi lled syringe with the plugged-up nee- Compliance is the reciprocal of elastance (elastic recoil), dle. You can pull the plunger out a small distance without much the ability to resist being deformed. Elastance also refers to the eff ort, but the cohesiveness of the water makes it diffi cult to pull ability of a body to return to its original shape when a deform- the plunger out any farther. Th e increased amount of work you ing force is removed. A lung that stretches easily (high compli- do trying to pull the plunger out is paralleled by the work your ance) has probably lost its elastic tissue and will not return to its inspiratory muscles must do when they contract during inspira- resting volume when the stretching force is released (low elas- tion. Th e bigger the breath, the more work is required to stretch tance). You may have experienced something like this with old the elastic lung. gym shorts. Aft er many washings the elastic waistband is easy By the end of a quiet inspiration, when the lungs are fully to stretch (high compliance) but lacking in elastance, making it expanded, intrapleural pressure falls to around -6 mm Hg impossible for the shorts to stay up around your waist. (Fig. 17.9 , point B2 ). During exercise or other powerful inspira- Analogous problems occur in the respiratory system. For tions, intrapleural pressure may reach -8 mm Hg or lower. example, as noted in the Running Problem, emphysema is a During expiration, the thoracic cage returns to its rest- disease in which elastin fi bers normally found in lung tissue are ing position. Th e lungs are released from their stretched posi- destroyed. Destruction of elastin results in lungs that exhibit tion, and the intrapleural pressure returns to its normal value of high compliance and stretch easily during inspiration. However, about - 3 mm Hg (point B3). Notice that intrapleural pressure these lungs also have decreased elastance, so they do not recoil never equilibrates with atmospheric pressure because the pleu- to their resting position during expiration. ral cavity is a closed compartment. To understand the importance of elastic recoil to expira- Pressure gradients required for air fl ow are created by the tion, think of an infl ated balloon and an infl ated plastic bag. work of skeletal muscle contraction. Normally, about 3–5% The balloon is similar to the normal lung. Its elastic walls of the body’s energy expenditure is used for quiet breathing. squeeze on the air inside the balloon, thereby increasing the During exercise, the energy required for breathing increases internal air pressure. When the neck of the balloon is opened substantially. Th e two factors that have the greatest infl uence to the atmosphere, elastic recoil causes air to flow out of on the amount of work needed for breathing are the stretch- the balloon. Th e infl ated plastic bag, on the other hand, is like the ability of the lungs and the resistance of the airways to air lung of an individual with emphysema. It has high compliance fl ow. and is easily infl ated, but it has little elastic recoil. If the infl ated

617 Mechanics of Breathing

plastic bag is opened to the atmosphere, most of the air remains much harder to infl ate. From this result, researchers concluded inside the bag. that lung tissue itself contributes less to resistance than once A decrease in lung compliance aff ects ventilation because thought. Some other property of the normal air-fi lled lung, a more work must be expended to stretch a stiff lung. Pathological property not present in the saline-fi lled lung, must create most conditions in which compliance is reduced are called restrictive of the resistance to stretch. lung diseases . In these conditions, the energy expenditure re- This property is the surface tension created by the thin quired to stretch less-compliant lungs can far exceed the normal fl uid layer between the alveolar cells and the air. At any air-fl uid work of breathing. Two common causes of decreased compli- interface, the surface of the fluid is under tension, like a thin ance are inelastic scar tissue formed in fi brotic lung diseases, and membrane being stretched. When the fl uid is water, surface ten- inadequate alveolar production of surfactant, a chemical that fa- sion arises because of the hydrogen bonds between water mol- cilitates lung expansion. ecules. Th e water molecules on the fl uid’s surface are attracted Pulmonary fi brosis is characterized by the development of to other water molecules beside and beneath them but are not stiff , fi brous scar tissue that restricts lung infl ation. In idiopathic attracted to gases in the air at the air-fl uid interface. pulmonary fibrosis {idios , one’s own}, the cause is unknown. Alveolar surface tension is similar to the surface tension Other forms of fi brotic lung disease result from chronic inhala- that exists in a spherical bubble, even though alveoli are not per- tion of fi ne particulate matter, such as asbestos and silicon, that fect spheres. Th e surface tension created by the thin fi lm of fl uid escapes the mucus lining the airways and reaches the alveoli. is directed toward the center of the bubble and creates pressure Wandering alveolar macrophages (see Fig. 17.2 g) then ingest in the interior of the bubble. Th e law of LaPlace is an expression the inhaled particulate matter. If the particles are organic, the of this pressure. It states that the pressure (P) inside a bubble macrophages can digest them with lysosomal enzymes. How- formed by a fl uid fi lm is a function of two factors: the surface ever, if the particles cannot be digested or if they accumulate tension of the fluid (T) and the radius of the bubble (r). This in large numbers, an infl ammatory process ensues. Th e macro- relationship is expressed by the equation phages then secrete growth factors that stimulate fi broblasts in P = 2T r the lung’s connective tissue to produce inelastic collagen. Pul- > monary fi brosis cannot be reversed. Notice in Figure 17.11 a that if two bubbles have diff erent diameters but are formed by fl uids with the same surface ten- sion, the pressure inside the smaller bubble is greater than that Surfactant Decreases the Work of Breathing inside the larger bubble. For years, physiologists assumed that elastin and other elastic How does this apply to the lung? In physiology, we can fibers were the primary source of resistance to stretch in the equate the bubble to a fluid-lined alveolus (although alveoli lung. However, studies comparing the work required to expand are not perfect spheres). Th e fl uid lining all the alveoli creates air-fi lled and saline-fi lled lungs showed that air-fi lled lungs are surface tension. If the surface tension (T) of the fl uid were the

LAW OF LaPLACE

(a) The two bubbles shown have the same surface tension (T). (b) Surfactant ( ) reduces surface tension (T). In the lungs, smaller According to the Law of LaPlace, pressure is greater in the alveoli have more surfactant, which equalizes the pressure smaller bubble. between large and small alveoli.

Law of LaPlace

P = 2T/r More surfactant decreases surface tension. Larger bubble Smaller bubble P = pressure r = 2 r = 1 T = surface tension r = 2 r = 1 T = 3 T = 3 r = radius T = 2 T = 1 P = (2 ϫ 3)/2 P = (2 ϫ 3)/1 According to the law of LaPlace, P = (2 ϫ 2)/2 P = (2 ϫ 1)/1 P = 3 P = 6 if two bubbles have the same P = 2 P = 2 surface tension, the smaller bubble will have higher pressure.

Fig. 17.11

618 Mechanics of Breathing same in small and large alveoli, small alveoli would have higher distress syndrome (NRDS) . In addition to having “stiff” (low- inwardly directed pressure than larger alveoli, and increased re- compliance) lungs, NRDS babies also have alveoli that collapse sistance to stretch. As a result, more work would be needed to each time they exhale. These infants must use a tremendous expand smaller alveoli. amount of energy to expand their collapsed lungs with each Normally, however, our lungs secrete a surfactant that re- breath. Unless treatment is initiated rapidly, about 50% of these duces surface tension. Surfactants (“ surf ace active age nts ”) are infants die. In the past, all physicians could do for NRDS ba- molecules that disrupt cohesive forces between water molecules bies was administer oxygen. Today, however, the prognosis for by substituting themselves for water at the surface. For example, NRDS babies is much better. Amniotic fl uid can be sampled to that product you add to your dishwasher to aid in the rinse cycle assess whether or not the fetal lungs are producing adequate is a surfactant that keeps the rinse water from beading up on amounts of surfactant. If they are not, and if delivery cannot the dishes (and forming spots when the water beads dry). In the be delayed, NRDS babies can be treated with aerosol adminis- lungs, surfactant decreases surface tension of the alveolar fl uid tration of artifi cial surfactant until the lungs mature enough to and thereby decreases resistance of the lung to stretch. produce their own. Th e current treatment also includes artifi cial Surfactant is more concentrated in smaller alveoli, making ventilation that forces air into the lungs ( positive-pressure venti- their surface tension less than that in larger alveoli (Fig. 17.11 b). lation ) and keeps the alveoli open. Lower surface tension helps equalize the pressure among alveoli of diff erent sizes and makes it easier to infl ate the smaller alve- 17 oli. With lower surface tension, the work needed to expand the Airway Diameter Determines alveoli with each breath is greatly reduced. Human surfactant Airway Resistance is a mixture containing proteins and phospholipids, such as di- The other factor besides compliance that influences the work palmitoylphosphatidylcholine , which are secreted into the alveo- of breathing is the resistance of the respiratory system to air lar air space by type II alveolar cells (see Fig. 17.2 g). flow. Resistance in the respiratory system is similar in many Normally, surfactant synthesis begins about the twenty- ways to resistance in the cardiovascular system. Th ree param- fi ft h week of fetal development under the infl uence of various eters contribute to resistance (R): the system’s length (L), the hormones. Production usually reaches adequate levels by the viscosity of the substance fl owing through the system ( h ) , a n d thirty-fourth week (about six weeks before normal delivery). the radius of the tubes in the system (r). As with fl ow in the car- Babies who are born prematurely without adequate concentra- diovascular system, Poiseuille’s law relates these factors to one tions of surfactant in their alveoli develop newborn respiratory another:

R r Lh r4 RUNNING PROBLEM > Edna has been experiencing shortness of breath while Because the length of the respiratory system is constant, exercising, so her physician runs some tests, including we can ignore L in the equation. Th e viscosity of air is almost measuring Edna’s lung volumes with spirometry. Part of the constant, although you may have noticed that it feels harder to test is a forced expiratory volume. With her lungs fi lled to breathe in a sauna fi lled with steam than in a room with normal their maximum with air, Edna is told to blow out as fast and humidity. Water droplets in the steam increase the viscosity of as forcefully as she can. The volume of air that Edna expels in the steamy air, thereby increasing its resistance to fl ow. Viscosity the fi rst second of the test (the forced expiratory volume in also changes slightly with atmospheric pressure, decreasing as one second, or FEV 1 ) is lower than normal because in COPD, airway resistance is increased. Another test the physician pressure decreases. A person at high altitude may feel less resis- orders is a complete blood count (CBC). The results of this test tance to air fl ow than a person at sea level. Despite these excep- show that Edna has higher-than-normal red blood cell count tions, viscosity plays a very small role in resistance to air fl ow. and hematocrit. Length and viscosity are essentially constant for the respi- ratory system. As a result, the radius (or diameter) of the air- Q4: When Edna fi lls her lungs maximally, the volume of air in ways becomes the primary determinant of airway resistance. her lungs is known as the capacity. When she Normally, however, the work needed to overcome resistance exhales all the air she can, the volume of air left in her of the airways to air fl ow is much less than the work needed to lungs is the . overcome the resistance of the lungs and thoracic cage to stretch. Q5: Why are Edna’s RBC count and hematocrit increased? Nearly 90% of airway resistance normally can be attributed (Hint: Because of Edna’s COPD, her arterial P is low.) to the trachea and bronchi, rigid structures with the smallest to- O2 tal cross-sectional area. Because these structures are supported by cartilage and bone, their diameters normally do not change, and their resistance to air fl ow is constant. However, accumu- lation of mucus from allergies or infections can dramatically

619 Mechanics of Breathing

increase resistance. If you have ever tried breathing through Concept Check Answers: End of Chapter your nose when you have a cold, you can appreciate how the narrowing of an upper airway limits air fl ow! 23. In a normal person, which contributes more to the work of breathing: Th e bronchioles normally do not contribute signifi cantly airway resistance or lung and chest wall elastance? to airway resistance because their total cross-sectional area is 24. Coal miners who spend years inhaling fi ne coal dust have much of their about 2000 times that of the trachea. Because the bronchioles alveolar surface area covered with scarlike tissue. What happens to are collapsible tubes, however, a decrease in their diameter can their lung compliance as a result? suddenly turn them into a significant source of airway resis- 25. How does the work required for breathing change when surfactant is tance. increases resistance to air fl ow and not present in the lungs? decreases the amount of fresh air that reaches the alveoli. Bronchioles, like arterioles, are subject to refl ex control by 26. A cancerous lung tumor has grown into the walls of a group of the nervous system and by hormones. However, most minute- bronchioles, narrowing their lumens. What has happened to the to-minute changes in bronchiolar diameter occur in response to resistance to air fl ow in these bronchioles? paracrines. Carbon dioxide in the airways is the primary paracrine 27. Name the neurotransmitter and receptor for parasympathetic that aff ects bronchiolar diameter. Increased CO2 in expired air re- bronchoconstriction. laxes bronchiolar smooth muscle and causes bronchodilation . Histamine is a paracrine that acts as a powerful broncho- constrictor. Th is chemical is released by mast cells in response to either tissue damage or allergic reactions. In severe allergic reactions, large amounts of histamine may lead to widespread bronchoconstriction and diffi cult breathing. Immediate medical Rate and Depth of Breathing Determine treatment in these patients is imperative. the Effi ciency of Breathing The primary neural control of bronchioles comes from T he efficiency of the heart is measured by the cardiac out- parasympathetic neurons that cause bronchoconstriction, a re- put, which is calculated by multiplying heart rate by stroke fl ex designed to protect the lower respiratory tract from inhaled volume. Likewise, we can estimate the effectiveness of ven- irritants. Th ere is no signifi cant sympathetic innervation of the tilation by calculating total pulmonary ventilation, the vol- bronchioles in humans. However, smooth muscle in the bronchi- ume of air moved into and out of the lungs each minute oles is well supplied with b -receptors that respond to epineph- 2 ( Fig. 17.12 a). Total pulmonary ventilation, also known as the rine. Stimulation of b -receptors relaxes airway smooth muscle 2 minute volume , is calculated as follows: and results in bronchodilation. This reflex is used therapeuti- cally in the treatment of asthma and various allergic reactions Total pulmonary ventilation = characterized by histamine release and bronchoconstriction. ventilation rate : tidal volume Table 17.1 summarizes the factors that alter airway resistance.

Table Factors That Aff ect Airway Resistance 17.1

Factor Aff ected by Mediated by

Length of the system Constant; not a factor

Viscosity of air Usually constant; humidity and altitude may alter slightly

Diameter of airways

Upper airways Physical obstruction Mucus and other factors

Bronchioles Bronchoconstriction Parasympathetic neurons (muscarinic receptors), histamine, leukotrienes

Bronchodilation Carbon dioxide, epinephrine

( b2 -receptors)

620 Fig. 17.12 ESSENTIALS

Ventilation

(a) Total pulmonary ventilation is greater than alveolar ventilation because of .

Total pulmonary ventilation Alveolar ventilation Alveolar ventilation is a better indication of how much fresh air reaches the Total pulmonary ventilation = ventilation rate × tidal volume (VT) alveoli. Fresh air remaining in the dead space does not get to the alveoli.

For example: 12 breaths/min × 500 mL breath = 6000 mL/min Alveolar ventilation = ventilation rate × (VT – dead space volume VD)

If dead space is 150 mL: 12 breaths/min × (500 – 150 mL) = 4200 mL/min

(b) Because the conducting airways do not exchange gases with the blood, they are known as anatomic dead space.

End of inspiration

150 1 At the end of inspiration, mL dead space is filled with fresh air.

2700 mL 2 Exhale 500 mL (tidal volume) Atmospheric 150 The first exhaled air comes air out of the dead space. Only 50 350 mL leaves the alveoli. 0 350

m L

Dead space is filled

with fresh air.

150

150 n E mL o x i p t

i a RESPIRATORY r

r

Only 350 mL of a

350 i

CYCLE IN t p

fresh air reaches i 2200 mL s

ADULT o n

alveoli. n 150 I

2200 mL The first 150 mL of air into the alveoli is stale air from the KEY Dead space filled dead space. with stale air P = 150 mm Hg (fresh air) O2 P ~ 100 mm Hg (stale air) O2 ~ 4 Inhale 500 mL 150 of fresh air mL (tidal volume).

2200 mL 3 At the end of expiration, the dead space is filled with “stale” air from alveoli.

End of expiration

FIGURE QUESTION Complete this table showing the effects of breathing pattern on alveolar ventilation. Assume dead space volume is 150 mL. Which pattern is the most efficient?

Tidal volume Ventilation rate Total pulmonary Fresh air to Alveolar ventilation (mL) (breaths/min) ventilation (mL/min) alveoli (mL) (mL/min)

500 (normal) 12 (normal) 6000 350 4200

300 (shallow) 20 (rapid)

750 (deep) 8 (slow)

621 Mechanics of Breathing

Th e normal ventilation rate for an adult is 12-20 breaths Using the same ventilation rate and tidal volume as before, (br) per minute. Using the average tidal volume (500 mL) and and a dead space of 150 mL, then the slowest ventilation rate, we get: Alveolar ventilation = = 12 br min : (500 – 150 mL br) = 4200 mL min Total pulmonary ventilation > > > 12 br min : 500 mL br = 6000 mL min = 6 L min > > > > Th us, at 12 breaths per minute, the alveolar ventilation is 4.2 L min. Although 6 L min of fresh air enters the respiratory Total pulmonary ventilation represents the physical move- > > system, only 4.2 L reaches the alveoli. ment of air into and out of the respiratory tract, but is it a good Alveolar ventilation can be drastically aff ected by changes in indicator of how much fresh air reaches the alveolar exchange the rate or depth of breathing, as you can calculate using the fi g- surface? Not necessarily. ure question in Figure 17.12 . Maximum voluntary ventilation , Some air that enters the respiratory system does not reach which involves breathing as deeply and quickly as possible, may the alveoli because part of every breath remains in the conduct- increase total pulmonary ventilation to as much as 170 L min. ing airways, such as the trachea and bronchi. Because the con- > Table 17.2 describes various patterns of ventilation, and ducting airways do not exchange gases with the blood, they are Table 17.3 gives normal ventilation values. known as the anatomic dead space. Anatomic dead space aver- ages about 150 mL. To illustrate the diff erence between the total volume of air Gas Composition in the Alveoli Varies that enters the airways and the volume of fresh air that reaches Little During Normal Breathing the alveoli, let’s consider a typical breath that moves 500 mL of air during a respiratory cycle ( Fig. 17.12 b). How much can a change in alveolar ventilation aff ect the amount of fresh air and oxygen that reach the alveoli? Figure 17.13 shows 1 At the end of an inspiration, lung volume is maximal, and how the partial pressures P a n d P in the alveoli vary with hy- O2 CO2 fresh air from the atmosphere fi lls the dead space. per- and hypoventilation. As alveolar ventilation increases above 2 Th e tidal volume of 500 mL is exhaled. However, the fi rst normal levels during hyperventilation, alveolar P increases, and O2 portion of this 500 mL to exit the airways is the 150 mL a l v e o l a r P falls. During hypoventilation, when less fresh air en- CO2 of fresh air that had been in the dead space, followed by ters the alveoli, alveolar P decreases and alveolar P increases. O2 CO2 350 mL of “stale” air from the alveoli. Even though 500 mL A dramatic change in alveolar ventilation pattern can aff ect of air exited the alveoli, only 350 mL of that volume left the gas partial pressures in the alveoli, but the P a n d P i n t h e O2 CO2 body. Th e remaining 150 mL of “stale” alveolar air stays in alveoli change surprisingly little during normal quiet breathing. the dead space. A l v e o l a r P is fairly constant at 100 mm Hg, and alveolar P O2 CO2 3 At the end of expiration, lung volume is at its minimum, stays close to 40 mm Hg. and stale air from the most recent expiration fi lls the ana- Intuitively, you might think that P would increase when O2 tomic dead space. fresh air fi rst enters the alveoli, then decrease steadily as oxy- 4 With the next inspiration, 500 mL of fresh air enters the gen leaves to enter the blood. Instead, we fi nd only very small airways. Th e fi rst air to enter the alveoli is the 150 mL of swings in P . Why? Th e reasons are that (1) the amount of oxy- O2 stale air that was in the anatomic dead space. Th e remain- gen that enters the alveoli with each breath is roughly equal to ing 350 mL of air to go into the alveoli is fresh air. Th e last the amount of oxygen that enters the blood, and (2) the amount 150 mL of inspired fresh air again remains in the dead of fresh air that enters the lungs with each breath is only a little space and never reaches the alveoli. more than 10% of the total lung volume at the end of inspiration. Th us, although 500 mL of air entered the alveoli, only 350 mL of that volume was fresh air. Th e fresh air entering the alveoli Concept Check Answers: End of Chapter equals the tidal volume minus the dead space volume. 28. If a person increases his tidal volume, what would happen to his alveolar P ? Because a signifi cant portion of inspired air never reaches O2 an exchange surface, a more accurate indicator of ventilation 29. If his breathing rate increases, what would happen to his alveolar P ? O2 effi ciency is alveolar ventilation , the amount of fresh air that reaches the alveoli each minute. Alveolar ventilation is calcu- lated by multiplying ventilation rate by the volume of fresh air Ventilation and Alveolar Blood Flow Are Matched that reaches the alveoli: Moving oxygen from the atmosphere to the alveolar exchange Alveolar ventilation = surface is only the fi rst step in external respiration. Next, gas ex- ventilation rate : (tidal volume – dead space) change must occur across the alveolar-capillary interface. Finally, blood fl ow ( ) past the alveoli must be high enough to

622 Mechanics of Breathing

Table Types and Patterns of Ventilation 17.2

Name Description Examples

Eupnea Normal quiet breathing

Hyperpnea Increased and/or volume Exercise in response to increased metabolism Hyperventilation Increased respiratory rate and/or Emotional hyperventilation; blowing volume without increased metabolism up a balloon Hypoventilation Decreased alveolar ventilation Shallow breathing; asthma; restrictive lung disease Tachypnea Rapid breathing; usually increased Panting respiratory rate with decreased depth 17 Dyspnea Difficulty breathing (a subjective feeling Various pathologies or hard exercise sometimes described as “air hunger”) Apnea Cessation of breathing Voluntary breath-holding; depression of CNS control centers

to recruit additional capillary beds during exercise is an example Normal Ventilation Values Table of the reserve capacity of the body. 17.3 in Pulmonary Medicine At the local level, the body attempts to match air fl ow and blood fl ow in each section of the lung by regulating the diam- Total pulmonary ventilation 6 L min > eters of the arterioles and bronchioles. Bronchiolar diameter is Total alveolar ventilation 4.2 L min mediated primarily by CO2 levels in exhaled air passing through > them ( Fig. 17.14 ). An increase in the P o f e x p i r e d a i r CO2 Maximum voluntary 125–170 L min causes bronchioles to dilate. A decrease in the P of expired > CO2 ventilation air causes bronchioles to constrict. Respiration rate 12–20 breaths min Although there is some autonomic innervation of pulmo- > nary arterioles, there is apparently little neural control of pul- monary blood fl ow. Th e resistance of pulmonary arterioles to pick up the available oxygen. Matching the ventilation rate into blood fl ow is regulated primarily by the oxygen content of the groups of alveoli with blood fl ow past those alveoli is a two-part interstitial fl uid around the arteriole. If ventilation of alveoli in process involving local regulation of both air fl ow and blood fl ow. one area of the lung is diminished, as shown in Figure 17.14 b, Alterations in pulmonary blood fl ow depend almost exclu- t h e P in that area decreases, and the arterioles respond by O2 sively on properties of the capillaries and on such local factors as constricting, as shown in Figure 17.14 c. Th is local vasoconstric- the concentrations of oxygen and carbon dioxide in the lung tis- tion is adaptive because it diverts blood away from the under- sue. Capillaries in the lungs are unusual because they are collaps- ventilated region to better-ventilated parts of the lung. ible. If the pressure of blood fl owing through the capillaries falls Note that constriction of pulmonary arterioles in response to below a certain point, the capillaries close off , diverting blood to low P is the opposite of what occurs in the systemic circulation. O2 pulmonary capillary beds in which blood pressure is higher. In the systemic circulation, a decrease in the P of a tissue causes O2 In a person at rest, some capillary beds in the apex (top) of local arterioles to dilate, delivering more oxygen-carrying blood to the lung are closed off because of low hydrostatic pressure. Cap- those tissues that are consuming oxygen. In the lungs, blood is pick- illary beds at the base of the lung have higher hydrostatic pres- ing up oxygen, so it does not make sense to send more blood to an sure because of gravity and thus remain open. Consequently, area with low tissue P due to poor ventilation. O2 blood fl ow is diverted toward the base of the lung. During exer- Another important point must be noted here. Local control cise, when blood pressure rises, the closed apical capillary beds mechanisms are not eff ective regulators of air and blood fl ow open, ensuring that the increased cardiac output can be fully oxy- under all circumstances. If blood flow is blocked in one pul- genated as it passes through the lungs. Th e ability of the lungs monary artery, or if air fl ow is blocked at the level of the larger

623 Mechanics of Breathing

sounds are more complicated to interpret than heart sounds, how- ALVEOLAR GASES ever, because breath sounds have a wider range of normal variation. As alveolar ventilation increases, alveolar P increases and P Normally, breath sounds are distributed evenly over the lungs O2 CO2 decreases. The opposite occurs as alveolar ventilation decreases. and resemble a quiet “whoosh” made by fl owing air. When air fl ow is reduced, such as in pneumothorax, breath sounds may be either Normal ventilation 4.2 L/min diminished or absent. Abnormal sounds include various squeaks, pops, wheezes, and bubbling sounds caused by fl uid and secretions in the airways or alveoli. Infl ammation of the pleural membrane Hypoventilation Hyperventilation 120 results in a crackling or grating sound known as a friction rub . It is caused by swollen, infl amed pleural membranes rubbing against each other, and it disappears when fl uid again separates them. 100 Diseases in which air fl ow is diminished because of increased P O2 airway resistance are known as obstructive lung diseases . When patients with obstructive lower airway diseases are asked to exhale ) in mm Hg 80 forcefully, air whistling through the narrowed airways creates a gas wheezing sound that can be heard even without a stethoscope. De- 60 pending on the severity of the disease, the bronchioles may even col- lapse and close off before a forced expiration is completed, reducing P CO2 both the amount and rate of air fl ow as measured by a spirometer. 40 Obstructive lung diseases include asthma, obstructive sleep apnea, emphysema, and chronic bronchitis. The latter two are sometimes called chronic obstructive pulmonary disease (COPD) 20 Alveolar partial pressure (P because of their ongoing, or chronic, nature. Obstructive sleep apnea { apnoia , breathless} results from obstruction of the upper airway, oft en due to abnormal relaxation of the muscles of the pharynx and 234 5678910 tongue that increases airway resistance during inspiration. Alveolar ventilation (L/min) Asthma is an inflammatory condition, often associated with allergies, that is characterized by bronchoconstriction and GRAPH QUESTION airway edema. Asthma can be triggered by exercise (exercise- What are the maximum alveolar P O2 induced asthma) or by rapid changes in the temperature or hu- and minimum P shown in this graph? CO2 midity of inspired air. Asthmatic patients complain of “air hunger” Fig. 17.13 and diffi culty breathing, or dyspnea . Th e severity of asthma at- tacks ranges from mild to life threatening. Studies of asthma at airways, local responses that shunt air or blood to other parts of the cellular level show that a variety of chemical signals may be the lung are ineff ective because in these cases no part of the lung responsible for inducing asthmatic bronchoconstriction. Among has normal ventilation or perfusion. these are acetylcholine, histamine, substance P (a neuropeptide), and leukotrienes secreted by mast cells, macrophages, and eosin- Concept Check Answers: End of Chapter ophils. Leukotrienes are lipid-like bronchoconstrictors that are released during the infl ammatory response. Asthma is treated 30. If a lung tumor decreases blood fl ow in one small section of the lung to with inhaled and oral medications that include b -adrenergic a minimum, what happens to P in the alveoli in that section and in 2 O2 the surrounding interstitial fl uid? What happens to P in that section? agonists, anti-infl ammatory drugs, and leukotriene antagonists. CO2 What is the compensatory response of the bronchioles in the aff ected section? Will the compensation bring ventilation in the aff ected section Concept Check Answers: End of Chapter of the lung back to normal? Explain. 31. Restrictive lung diseases decrease lung compliance. How will inspiratory reserve volume change in patients with a restrictive lung disease? 32. Chronic obstructive lung disease causes patients to lose the ability to Auscultation and Spirometry Assess exhale fully. How does residual volume change in these patients? Pulmonary Function Most pulmonary function tests are relatively simple to perform. Th is completes our discussion of the mechanics of ventilation. Auscultation of breath sounds is an important diagnostic tech- RUNNING PROBLEM CONCLUSION nique in pulmonary medicine, just as auscultation of heart sounds is an important technique in cardiovascular diagnosis. Breath

624 Mechanics of Breathing

Local control mechanisms attempt to match ventilation and perfusion.

(a) Normally perfusion of blood past alveoli is matched to (b) Ventilation-perfusion mismatch caused by alveolar ventilation to maximize gas exchange. under-ventilated alveoli.

Arteriole If ventilation decreases in a group of alveoli, Bronchiole Low P increases and P CO2 O2 oxygen decreases. Blood blood flowing past those alveoli does not get oxygenated. PCO Alveoli Alveoli 2 P O2

17

(c) Local control mechanisms try to keep ventilation (d) Bronchiole diameter is mediated primarily by CO2 levels in and perfusion matched. exhaled air passing through them.

Decreased tissue P O2 around underventilated Local Control of Arterioles and Bronchioles by alveoli constricts their Oxygen and Carbon Dioxide arterioles, diverting blood to better ventilated alveoli. Gas Bronchioles Pulmonary Systemic composition arteries arteries

P increases Dilate (Constrict)* Dilate CO2

P decreases CO2 Constrict (Dilate) Constrict

P increases (Constrict) (Dilate) Constrict O2

P decreases (Dilate) Constrict Dilate O2

Blood flow diverted * Parentheses indicate weak responses. to better ventilated alveoli

FIGURE QUESTIONS Bronchiole _____? A blood clot prevents gas exchange in a group of alveoli.

1. What happens to tissue and alveolar gases? ? Tissue PO _____ 2. What do bronchioles and arterioles do 2 in response? ? ? P Arteriole _____ O2 ? P CO2

Blood clots prevent gas exchange. Fig. 17.14

625 Mechanics of Breathing

RUNNING PROBLEM CONCLUSION

Emphysema Edna leaves the offi ce with prescriptions for a mucus- (www.lungusa.org ), COPD is the fourth leading cause of thinning drug, a bronchodilator, and anti-infl ammatory death in the United States and costs more than $30 billion drugs to keep her airways as open as possible. She has per year in direct medical costs and indirect costs such as agreed to try to stop smoking once more and also has a lost wages. prescription and brochures for that. Unfortunately, the In this running problem you learned about chronic lung changes that take place with COPD are not reversible, obstructive pulmonary disease. Now check your and Edna will require treatment for the rest of her life. understanding of the physiology in the problem by According to the American Lung Association comparing your answers with those in the following table.

Question Facts Integration and Analysis

1. What does narrowing of the air- The relationship between tube radius and When resistance increases, the body must ways do to the resistance airways resistance is the same for air fl ow as for use more energy to create air fl ow. off er to air fl ow? blood fl ow: as radius decreases, resistance increases.

2. Why do people with chronic bron- Cigarette smoke paralyzes the cilia that Bacteria trapped in the mucus can multiply chitis have a higher-than-normal rate sweep debris and mucus out of the airways. and cause respiratory infections. of respiratory infections? Without the action of cilia, mucus and trapped particles pool in the airways.

3. Name the muscles that patients Normal passive expiration depends on Forceful expiration involves the internal with emphysema use to exhale elastic recoil of muscles and elastic tissue in intercostal muscles and the abdominal actively. the lungs. muscles.

4. When Edna fi lls her lungs maxi- The maximum volume of air in the lungs is N/A mally, the volume of air in her lungs the total lung capacity. Air left in the lungs is known as the capacity. after maximal exhalation is the residual When she exhales all the air she can, volume. the volume of air left in her lungs is the .

5. Why are Edna’s RBC count and Because of Edna’s COPD, her arterial P is Low arterial oxygen levels trigger EPO re- O2 hematocrit increased? low. The major stimulus for red blood cell lease, which increases the synthesis of red synthesis is . blood cells. More RBCs provide more bind- ing sites for oxygen transport.

Test your understanding with: • Practice Tests • PhysioExTM Lab Simulations • Running Problem Quizzes • Interactive Physiology TM • A&PFlix Animations Animations www.masteringaandp.com

626 Mechanics of Breathing

Chapter Summary

Air fl ow into and out of the lungs is another example of the principle from changes in the diameter of the tubes through which it fl ows. Th e of mass fl ow. Like blood fl ow, air fl ow is bulk fl ow that requires a pump mechanical properties of the pleural sacs and elastic recoil in the chest to create a pressure gradient and that encounters resistance, primarily wall and lung tissue are essential for normal ventilation.

1. Aerobic metabolism in living cells consumes oxygen and produces 15. Boyle’s law states that as the volume available to a gas increases, carbon dioxide.) the gas pressure decreases. Th e body creates pressure gradients by 2. Gas exchange requires a large, thin, moist exchange surface; a pump changing thoracic volume. ( Fig. 17.6 b) to move air; and a circulatory system to transport gases to the cells. 3. Respiratory system functions include gas exchange, pH regulation, Ventilation vocalization, and protection from foreign substances. Respiratory:Respiratory: PulmonaryPulmonary VentilationVentilation The Respiratory System 16. A single respiratory cycle consists of an inspiration and an expiration. Respiratory:Respiratory: AnatomyAnatomy RevieRevieww 17 17. Tidal volume is the amount of air taken in during a single normal 4. Cellular respiration refers to cellular metabolism that consumes inspiration. Vital capacity is tidal volume plus expiratory and inspi- oxygen. External respiration is the exchange of gases between the at- ratory reserve volumes . Air volume in the lungs at the end of maxi- mosphere and cells of the body. It includes ventilation, gas exchange mal expiration is the residual volume . ( Fig. 17.7 b) at the lung and cells, and tra nsport of gases in the blood. Ventila- 18. Air flow in the respiratory system is directly proportional to the tion is the movement of air into and out of the lungs. ( Fig. 17.1 ) pressure gradient, and inversely related to the resistance to air fl ow 5. Th e respiratory system consists of anatomical structures involved in off ered by the airways. ventilation and gas exchange. 19. During inspiration , alveolar pressure decreases, and air fl ows into 6. The upper respiratory tract includes the mouth, nasal cavity, the lungs. Inspiration requires contraction of the inspiratory mus- pharynx , and larynx . Th e lower respiratory tract includes the tra- cles and the diaphragm. ( Fig. 17.9 ) chea, bronchi , bronchioles, and exchange surfaces of the alveoli . 20. Expiration is usually passive, resulting from elastic recoil of the ( Fig. 17.2 b) lungs. 7. The thoracic cage is bounded by the ribs, spine, and diaphragm . 21. Active expiration requires contraction of the internal intercostal Two sets of intercostal muscles connect the ribs. ( Fig. 17.2 a) and abdominal muscles. 8. Each lung is contained within a double-walled pleural sac that con- 22. Intrapleural pressures are subatmospheric because the pleural cav- tains a small quantity of pleural fl uid . ( Figs. 17.2 c, 17.3 ) ity is a sealed compartment. ( Figs. 17.9 , 17.10 ) 9. Th e two primary bronchi enter the lungs. Each primary bronchus 23. Compliance is a measure of the ease with which the chest wall and divides into progressively smaller bronchi and fi nally into collaps- lungs expand. Loss of compliance increases the work of breathing. ible bronchioles . ( Figs. 17.2 e, 17.4 ) Elastance is the ability of a stretched lung to return to its normal 10. Th e upper respiratory system fi lters, warms, and humidifi es inhaled volume. air. 24. Surfactant decreases surface tension in the fl uid lining the alveoli. 11. Th e alveoli consist mostly of thin-walled type I alveolar cells for gas Reduced surface tension prevents smaller alveoli from collapsing exchange. Type II alveolar cells produce surfactant. A network of and also makes it easier to infl ate the lungs. ( Fig. 17.11 ) capillaries surrounds each alveolus. (Fig. 17.2 f, g) 25. Th e diameter of the bronchioles determines how much resistance 12. Blood fl ow through the lungs equals cardiac output. Resistance to they off er to air fl ow. blood fl ow in the pulmonary circulation is low. Pulmonary arterial 26. Increased CO2 in expired air dilates bronchioles. Parasympathetic pressure averages 25 8 mm Hg. neurons cause bronchoconstriction in response to irritant stimuli. > Th ere is no signifi cant sympathetic innervation of bronchioles, but Gas Laws epinephrine causes bronchodilation . ( Tbl. 17.1 ) 27. Total pulmonary ventilation = tidal volume : ventilation rate. = : Respiratory:Respiratory: PulmonaryPulmonary VentilationVentilation Alveolar ventilation ventilation rate (tidal volume – dead space volume). ( Fig. 17.12 a) 13. Th e total pressure of a mixture of gases is the sum of the pressures of 28. Alveolar gas composition changes very little during a normal respi- the individual gases in the mixture (Dalton’s law ). Partial pressure is ratory cycle. Hyperventilation increases alveolar P and decreases O2 the pressure contributed by a single gas in a mixture. (Fig. 17.6 ) alveolar P . Hypoventilation has the opposite eff ect. ( Fig. 17.13 ) CO2 14. Bulk fl ow of air occurs down pressure gradients, as does the move- 29. Local mechanisms match air fl ow and blood fl ow around the alveoli. ment of any individual gas making up the air. Increased levels of CO2 dilate bronchioles, and decreased O2 con- stricts pulmonary arterioles. ( Fig. 17.14 )

627 Mechanics of Breathing

Questions

Level One Reviewing Facts and Terms Level Two Reviewing Concepts 1. List four functions of the respiratory system. 15. Compare and contrast the terms in each of the following sets: 2. Give two defi nitions for the word respiration. (a) compliance and elastance 3. Which sets of muscles are used for normal quiet inspiration? For (b) inspiration, expiration, and ventilation normal quiet expiration? For active expiration? What kind(s) of (c) intrapleural pressure and alveolar pressure muscles are the diff erent respiratory muscles (skeletal, cardiac, or (d) total pulmonary ventilation and alveolar ventilation smooth)? (e) type I and type II alveolar cells 4. What is the function of pleural fl uid? (f) pulmonary circulation and systemic circulation 5. Name the anatomical structures that an oxygen molecule passes on 16. List the major paracrines and neurotransmitters that cause bron- its way from the atmosphere to the blood. choconstriction and bronchodilation. What receptors do they act through? (muscarinic, nicotinic, a , b , b ) 6. Diagram the structure of an alveolus, and state the function of each 1 2 part. How are capillaries associated with an alveolus? 17. Compile the following terms into a map of ventilation. Use up ar- rows, down arrows, greater than symbols ( 7 ), and less than symbols 7. Trace the path of the pulmonary circulation. About how much (6 ) as modifi ers. You may add other terms. blood is found here at any given moment? What is a typical arterial blood pressure for the pulmonary circuit, and how does this pres- • abdominal muscles • inspiratory muscles sure compare with that of the systemic circulation? • air fl ow • internal intercostals 8. What happens to inspired air as it is conditioned during its passage • contract • PA through the airways? • diaphragm • Patm • expiratory muscles • P 9. During inspiration, most of the thoracic volume change is the result intrapleural • external intercostals • quiet breathing of movement of the . • forced breathing • relax 10. Describe the changes in alveolar and intrapleural pressure during • in, out, from, to • scalenes one respiratory cycle. 18. Decide whether each of the following parameters will increase, de- 11. What is the function of surfactants in general? In the respiratory crease, or not change in the situations given. system? (a) airway resistance with bronchodilation 12. Of the three factors that contribute to the resistance of air flow (b) intrapleural pressure during inspiration through a tube, which plays the largest role in changing resistance (c) air fl ow with bronchoconstriction in the human respiratory system? (d) bronchiolar diameter with increased P CO2 13. Match the following items with their correct effect on the (e) tidal volume with decreased compliance bronchioles: (f) alveolar pressure during expiration (a) histamine 1. bronchoconstriction 19. D e fi ne the following terms: pneumothorax, spirometer, ausculta- (b) epinephrine 2. bronchodilation tion, hypoventilation, bronchoconstriction, minute volume, partial pressure of a gas. (c) acetylcholine 3. no eff ect 20. Th e cartoon coyote is blowing up a balloon in another attempt to (d) increased P CO2 catch the roadrunner. He first breathes in as much air as he can, 14. Refer to the spirogram in the fi gure below: then blows out all he can into the balloon. (a) Th e volume of air in the balloon is equal to the of 4 the coyote’s lungs. This volume can be measured directly by measuring the balloon volume or by adding which respiratory 3 volumes together? (b) In 10 years, when the coyote is still chasing the roadrunner, will Volume 2 he still be able to put as much air into the balloon in one breath? (liters) Explain. 21. Match the descriptions to the appropriate phase(s) of ventilation: 1 (a) usually depend(s) on elastic recoil 1. inspiration

0 (b) is/are easier when lung 2. expiration 15 sec compliance decreases Time (c) is/are driven mainly by positive 3. both inspiration and (a) Label tidal volume ( VT), inspiratory and expiratory reserve intrapleural pressure generated expiration volumes (IRV and ERV), residual volume (RV), vital capacity by muscular contraction (VC), total lung capacity (TLC). (d) is usually an active process 4. neither (b) What is the value of each of the volumes and capacities you requiring smooth muscle labeled? contraction (c) What is this person’s ventilation rate?

628 Mechanics of Breathing

22. Draw and label a graph showing the P of air in the primary bron- 28. You have a mixture of gases in dry air, with an atmospheric pressure O2 chi during one respiratory cycle. ( Hint: What parameter goes on of 760 mm Hg. Calculate the partial pressure of each gas if the com- each axis?) position of the air is: 23. Lung compliance increases but chest wall compliance decreases as (a) 21% oxygen, 78% nitrogen, 0.3% carbon dioxide we age. In the absence of other changes, would the following param- (b) 40% oxygen, 13% nitrogen, 45% carbon dioxide, 2% hydrogen eters increase, decrease, or not change as compliance decreases? (c) 10% oxygen, 15% nitrogen, 1% argon, 25% carbon dioxide (a) work required for breathing 29. Li is a tiny woman, with a tidal volume of 400 mL and a respiratory (b) ease with which lungs infl ate rate of 12 breaths per minute at rest. What is her total pulmonary (c) lung elastance ventilation? Just before a physiology exam, her ventilation increases (d) airway resistance during inspiration to 18 breaths per minute from nervousness. Now what is her to- 24. Will increase, decrease, or not change the tal pulmonary ventilation? Assuming her anatomic dead space is following? 120 mL, what is her alveolar ventilation in each case? (a) work required for breathing 30. You collected the following data on your classmate Neelesh: (b) lung compliance Minute volume = 5004 mL min > (c) surface tension in the alveoli Respiratory rate = 3 breaths 15 sec > Vital capacity = 4800 mL Level Three Problem Solving Expiratory reserve volume = 1000 mL 25. Assume a normal female has a resting tidal volume of 400 mL, a What are Neelesh’s tidal volume and inspiratory reserve volume? respiratory rate of 13 breaths min, and an anatomic dead space of 31. Use the fi gure below to help solve this problem. A spirometer with 17 > 125 mL. When she exercises, which of the following scenarios would a volume of 1 liter ( V1) is fi lled with a mixture of oxygen and he- lium, with the helium concentration being 4 g L ( C ). Helium does be most effi cient for increasing her oxygen delivery to the lungs? > 1 (a) increase respiratory rate to 20 breaths min but have no change not move from the lungs into the blood or from the blood into > in tidal volume the lungs. A subject is told to blow out all the air he possibly can. (b) increase tidal volume to 550 mL but have no change in res- Once he fi nishes that exhalation, his lung volume is V2. He then piratory rate puts the spirometer tube in his mouth and breathes quietly for sev- (c) increase tidal volume to 500 mL and respiratory rate to eral breaths. At the end of that time, the helium is evenly dispersed 15 breaths min in the spirometer and the subject’s lungs. A measurement shows > the new concentration of helium is 1.9 g L. What was the subject’s Which of these scenarios is most likely to occur during exercise in > lung volume at the start of the experiment? (Hint: C V = C V ) real life? 1 1 2 2 26. A 30-year-old computer programmer has had asthma for 15 years. Helium/O When she lies down at night, she has spells of wheezing and cough- 2 mixture V1 ing. Over the years, she has found that she can breathe better if she sleeps sitting nearly upright. Upon examination, her doctor fi nds that she has an enlarged thorax. Her lungs are overinflated on X-ray. Here are the results of her examination and pulmonary func- tion tests. Use the normal values and abbreviations in Figure 17.8 to help answer the questions. Ventilation rate: 16 breaths min > Tidal volume: 600 mL ERV: 1000 mL RV: 3500 mL 32. Th e graph shows one lung under two diff erent conditions, A and B. Inspiratory capacity: 1800 mL What does this graph show? (a) the effect of lung volume on Vital capacity: 2800 mL pressure, or (b) the effect of pressure on lung volume? In which Functional residual capacity: 4500 mL condition does the lung have higher compliance, or is compliance TLC: 6300 mL the same in the two situations? Aft er she is given a bronchodilator, her vital capacity increased to 3650 mL. A B (a) What is her minute volume? (b) Explain the change in vital capacity with bronchodilators. (c) Which other values are abnormal? Can you explain why they might be, given her history and fi ndings? Volume

Level Four Quantitative Problems 27. A container of gas with a movable piston has a volume of 500 mL and a pressure of 60 mm Hg. Th e piston is moved, and the new pres- sure is 150 mm Hg. What is the new volume of the container? Pressure

629 Mechanics of Breathing

Answers

22. Th e knife wound would collapse the left lung if the knife punctured Answers to Concept Check Questions the pleural membrane. Loss of adhesion between the lung and chest wall would release the inward pressure exerted on the chest 1. Cellular respiration is intracellular and uses O2 and organic sub- wall, and the rib cage would expand outward. Th e right side would strates to produce ATP. External respiration is exchange and trans- be unaff ected as the right lung is contained in its own pleural sac. port of gases between the atmosphere and cells. 23. Normally, lung and chest wall elastance contribute more to the 2 . Th e upper respiratory tract includes the mouth, nasal cavity, phar- work of breathing. ynx, and larynx. Th e lower respiratory tract includes the trachea, 24. Scar tissue reduces lung compliance. bronchi, bronchioles, and exchange surface of lungs. 25. Without surfactant, the work of breathing increases. 3. Velocity is highest in the trachea and lowest in the bronchioles. 26. When bronchiolar diameter decreases, resistance increases. 4. Pleural fl uid reduces friction and holds the lungs tight against the chest wall. 27. Neurotransmitter is acetylcholine, and receptor is muscarinic. 28. Increased tidal volume increases alveolar P . 5 . Th e thoracic cage consists of the rib cage with intercostal muscles, O2 29. Increased breathing rate increases alveolar P . Increasing breath- spinal (vertebral) column, and diaphragm. The thorax contains O2 two lungs in pleural sacs, the heart and pericardial sac, esophagus, ing rate or tidal volume increases alveolar ventilation. and major blood vessels. 30. P in alveoli in the aff ected section will increase because O is not O2 2 leaving the alveoli. P will decrease because new CO is not en- 6 . Th e bronchioles are collapsible. CO2 2 tering the alveoli from the blood. Bronchioles constrict when P 7. If cilia cannot move mucus, the mucus collecting in the airways CO2 triggers a cough refl ex to clear out the mucus. decreases (see Fig. 17.14 ), shunting air to areas of the lung with better blood fl ow. Th is compensation cannot restore normal ven- 8. Blood fl ow is approximately equal in the pulmonary trunk and aorta. tilation in this section of lung, and local control is insuffi cient to (Normally some venous blood leaving the bronchi, pleura, and part maintain homeostasis. of the heart bypasses the pulmonary circulation and drains directly into the left side of the heart. Th is is called an anatomic shunt.) 31. Inspiratory reserve volume decreases. 9. Increased hydrostatic pressure causes greater net fi ltration out of 32. Residual volume increases in patients who cannot fully exhale. capillaries and may result in pulmonary edema. 10. Mean pressure = 8 mm Hg + 1 3(25 - 8) mm Hg = 8 + 1 7 3 mm > > Answers to Figure and Graph Questions Hg = 13.7 mm Hg. : = 11. 720 mm Hg 0.78 562 mm Hg Figure 17.9 : 1. Alveolar pressure is greatest in the middle of 12. 700 mm Hg - 47 mm Hg = 653 mm Hg : 21% = 137.1 mm Hg P O2 expiration and least in the middle of inspiration. It is equal to atmo- 13. Lung capacities are the sum of two or more lung volumes. spheric pressure at the beginning and end of inspiration and expiration. 14. Residual volume cannot be measured directly. 2. When lung volume is at its minimum, alveolar pressure is (c) 15. If aging individuals have reduced vital capacity while total lung ca- moving from maximum to minimum and external intercostal muscle contraction is (b) minimal. 3. 2 breaths 8 sec = ? breaths 60 pacity does not change, then residual volume must increase. > > sec = 15 breaths min. 16. As air becomes humidifi ed, the P decreases. > O2 Figure 17.12 : Shallow and rapid: total pulmonary ventilation = 17. Air fl ow reverses direction during a respiratory cycle, but blood 6000 mL min, 150 mL fresh air, alveolar ventilation = 3000 mL fl ows in a loop and never reverses direction. > > min. Slow and deep: total pulmonary ventilation = 6000 mL min, > 18. See Figures 17.2 c and 17.3 . Th e lungs are enclosed in a pleural sac. 600 mL fresh air, alveolar ventilation = 4800 mL min. Slow and > One pleural membrane attaches to the lung, and the other lines the deep is the most effi cient. thoracic cage. Pleural fl uid fi lls the pleural sac. Figure 17.13 : Alveolar P goes to 120 mm Hg and P falls to O2 CO2 19. Scarlett will be more successful if she exhales deeply, as this will de- about 19 mm Hg. crease her thoracic volume and will pull her lower rib cage inward. Figure 17.14 : 1. Alveolar P increases and P decreases in the O2 CO2 20. Inability to cough decreases the ability to expel the potentially aff ected alveoli. Local tissue P increases. 2. Th is constricts local O2 harmful material trapped in airway mucus. arterioles, which then shunts blood to better-perfused sections of 21. A hiccup causes a rapid decrease in both intrapleural pressure and lung. Bronchioles constrict to divert air to better-perfused alveoli. alveolar pressure.

630 Mechanics of Breathing

Answers to Review Questions

Level One Reviewing Facts and Terms 19. Pneumothorax—air in the pleural cavity. Spirometer—device used to measure ventilation. Auscultation—listening for body sounds. Hypoventila- 1. gas exchange, vocalization, pH regulation, and protection tion—decreased pulmonary ventilation. Bronchoconstriction—decrease in 2. Cellular respiration—oxygen and nutrients are used for energy production. bronchiole radius. Minute volume—total pulmonary ventilation. Partial External respiration—gas exchange between atmosphere and cells. pressure of gas—portion of total pressure in a mixture of gases that is con- 3. Quiet inspiration—external intercostals, scalenes, and diaphragm. Quiet tributed by a specific gas. expiration—no significant muscle contraction. Active expiration—internal 20. (a) vital capacity . Sum of tidal volume and expiratory and inspiratory reserve intercostals and abdominal muscles. These are all skeletal muscles. volumes. (b) No, because lung function decreases with age as elastance and 4. Lubrication between lungs and internal thoracic surface compliance diminish. 5. Nose and mouth, pharynx, larynx, trachea, main bronchus, secondary bron- 21. (a) 2, (b) 2, (c) 4, (d) 4 chi, bronchioles, epithelium of the alveoli, interstitial fluid, and capillary 22. x -axis—time; y -axis— P . During inspiration, the P of the primary bron- endothelium O2 O2 chi will increase, as fresh air P = 160 mm Hg) replaces the stale air 6. See Figure 17.2 g and h. Type I—gas exchange; type II—surfactant. Mac- 1 O2 P = 100 mm Hg . During expiration, the P will decrease, as the oxygen- rophages ingest foreign material. Capillary endothelium is almost fused to 1 O2 2 O2 depleted air exits the alveoli. The curve will vary from 100 mm Hg to 160 mm Hg. the alveolar epithelium, and the space between alveoli is almost filled with capillaries. 23. (a) Work increases. (b) Lungs inflate more easily. (c) Elastance decreases. (d) Airway resistance is not affected. 7. Right ventricle to pulmonary trunk, to left and right pulmonary arteries, 17 smaller arteries, arterioles, capillaries, venules, small veins, pulmonary 24. (a) decrease (b) increase (c) decrease veins, left atrium. Contains about 0.5 L of blood. Arterial pressure is 25 8, > compared with 120 80 for systemic. Level Three Problem Solving > 8. Warmed, humidified, and cleaned (filtered) 25. Resting alveolar ventilation = 3575 mL min. Exercising: (a) 5500 mL min > > (b) 5525 mL min (c) 5625 mL min. Increasing both rate and depth has the 9. diaphragm > > largest effect and is what would happen in real life. 10. See Figure 17.9 . 26. (a) 9600 mL min. (b) Dilating bronchioles reduces airway resistance. The 11. Surfactant decreases surface tension of water and makes it easier for lungs to > patient is able to force more air out of the lungs on expiration, which in- inflate and stay inflated. creases her ERV and decreases her RV. (c) Respiratory rate is normal, but 12. radius of the airways lung volumes are abnormal. Her high RV is confirmed by the X-ray. In 13. (a) 1 (b) 2 (c) 1 (d) 2 obstructive lung diseases such as asthma, the bronchioles collapse on expira- 14. (a) See Figure 17.7 . (b) V = 0.5 L, IRV = 1.25 L, ERV = 1.0 L. (c) 3 breaths tion, trapping air in the lungs and resulting in hyperinflation. Her low IRV T > 15 sec * 60 sec min = 12 br min accounts for most of the low vital capacity and is to be expected in someone > > with asthma, where the lungs are already overinflated at the beginning of inspiration. Her higher tidal volume may be the result of the effort she must Level Two Reviewing Concepts exert to breathe. 15. (a) Compliance—ability to deform in response to force; elastance—abil- ity to resume original shape after deforming force has been removed. Level Four Quantitative Problems (b) Ventilation—air exchange between atmosphere and lungs. Inspira- = = tion—air movement into lungs. Expiration—air movement out of lungs. 27. P1V1 P2V2. New volume 200 mL = = = = (c) Intrapleural pressure—always subatmospheric (except during forced 28. (a) O2 160 mm Hg, nitrogen 593 mm Hg, CO2 2.3 mm Hg. (b) O2 304 = = = = expiration, when it may become positive); alveolar pressures vary from mm Hg, nitrogen 99 mm Hg, CO2 342 mm Hg, H2 15 mm Hg. (c) O2 = = = subatmospheric to above atmospheric. (d) Total pulmonary ventilation— 76 mm Hg, nitrogen 114 mm Hg, argon 8 mm Hg, CO2 190 mm Hg. volume of air entering or leaving airways in a given period of time. Alveo- 29. Total pulmonary ventilation = 4800 mL min. Before an exam, ventilation > lar ventilation—volume of air entering or leaving alveoli in a given period is 7200 mL min. Alveolar ventilation is 3360 mL min (at rest) and 5040 > > of time. (e) Type I—thin cells for gas exchange; Type II—synthesize and mL min (before exam). > secrete surfactant. (f) Pulmonary—from right heart to lung and back to 30. Tidal volume = 417 mL/breath. IRV = 3383 mL left atrium. Systemic—left heart to most tissues and back to right atrium. 31. Lung volume is 1.1 L. (Did you forget to subtract the volume of the 16. Bronchoconstrictors: histamine, leukotrienes, acetylcholine (muscarinic); spirometer?) bronchodilators: carbon dioxide, epinephrine b 1 22 32. (b). The lung in A has the highest compliance 17. See Figs. 17.8 and 17.9 . 18. (a) decrease (b) decrease (c) decrease (d) increase (e) decrease (f) increase

Photo Credits

CO: Biophoto Associates/Photo Researchers, Inc.

631 632